Где-то в альтернативной вселенной, согласно MWI, я стал гениальным физиком. Но в этой вселенной просто подробно слежу за профессиональными публикациями в области физики, зарабатывая себе на хлеб развозкой пиццы базами данных. Как следствие чуть более глубоких знаний, я не могу смотреть никаких научно-популярных передач. Это как железом по стеклу — тут oversimplification, тут просто неверно, тут есть тонкости…

Я решил сформировать список наиболее часто встречающихся неправд и полуправд. Итак, самое частое это…

Большой взрыв


Его изображают примерно так:



Далее обычно следует заявление: «Когда Вселенная была не больше размера атома...» итд. Здесь пропущено самое главное слово, которое полностью меняет смысл: «Когда видимая Вселенная была не больше размера атома...». Видимая Вселенная, грубо говоря, это область, где свет мог пройти от одного места до другого с момента t=0. То есть небольшой шарик, который раздувается (примерно!) со скоростью света (на самом деле быстрее). Таких шариков может быть бесконечно много.

Более того, если Вселенная бесконечна, то она бесконечна с самого начала. Бесконечна и почти однородна. И эта картинка (и еще красивый рендер со взрывом, который любят показывать) никакого отношения к реальности не имеет. Нет никакой точки, где был взрыв, который распространялся в окружающую пустоту. Повторюсь, вселенная была почти однородной (и чем дальше в прошлое, тем более однородной).

Кроме того, теория Большого взрыва это НЕ теория о большом взрыве (t=0). Это теория о том, что случилось после большого взрыва (t>0). Она не отвечает на вопрос, в частности, почему этот большой взрыв произошел. Да и само название — Big Bang — вначале использовалось как высмеивание теории, но прилипло к ней.

Черные дыры


О, здесь просто бездна (черная дыра) популярных мифов. Начнем с Интерстеллар:



Для того, чтобы замедление времени было бы таким, планета должна была быть на расстоянии чуть более 0.003% от радиуса Шварцшильда. Между тем, судя по виду на черную дыру:



планета находится от горизонта событий не менее чем на 10*Rs. Мы уже не говорим о том, что такая планета, наверное, худший кандидат на планету, куда стоило лететь. Конечно, это художественный фильм. Никто не судит Звездные войны за звук взрывов в космосе. Но Звездные войны никогда и не претендовали на научность. А Интерстеллар — претендовал, причем с пафосом. Поэтому с него и другой спрос.

Чтобы закрыть тему с Интерстеллар, по современным научным данным, библиотеки внутри черной дыры тоже нет.

Замедление времени в черной дыре бесконечно


Интересный случай полуправды и неверных выводов. Начинается все с правильного утверждения: при приближении к горизонту событий для наблюдателей, подвешенных над черной дырой, замедление времени стремится к бесконечности. Это верно. Но из этого утверждения вырастает сразу несколько мифов:

  • Замедление времени для падающего в черную дыру наблюдателя бесконечно
  • Поэтому он увидит все будущее Вселенной — она для него будет ускоряться
  • Падающие наблюдатели так и висят где-то недалеко от горизонта — ведь время для них бесконечно замедлилось
  • Можно броситься им вслед и догнать их где-то там у горизонта, сколько бы ни прошло времени
  • Более того, они так никогда и не упадут в черную дыру, ведь для этого нужно бесконечное время
  • Более того, можно даже сказать что на самом деле черных дыр не существует, ведь для их формирования нужно бесконечное время.

Все эти выводы растут из первого, неверного. Но он выглядит так правдоподобно, что в начале прошлого века долго вводил в заблуждение даже профессиональных физиков! Итак, мы имеем набор точек все ближе и ближе к горизонту, и замедление времени при приближении к горизонту стремится к бесконечности, а раз траектория падающего в черную дыру тела проходит через все эти точки, то для него замедление времени стремится к бесконечности! Логично? А вот и нет.

Замедление времени — это свойство траектории, а не точки пространства. То, что верно для наблюдателя, «подвешенного» на фиксированном растоянии над горизонтом, неверно для наблюдателя, падающего в черную дыру. Собственно, долгое время даже сам Эйнштейн не мог решить уравнение своей же общей теории относительности для областей внутри горизонта черной дыры — все упиралось в бесконечности на горизонте событий. Прогресс появился, когда вместо точек пространства стали рассматривать систему «дождя», свободно падающего внутрь черной дыры. В этой системе бесконечностей на горизонте не появлялось.

Более того, стало ясно то, что позже назвали «No drama» — при пересечении горизонта событий ничего особенного для свободно падающего наблюдателя не происходит — никакой тревожной музыки, пиу-пиу и зеленоватых всполохов. Провал под горизонт можно даже не заметить!

Для тех, кто хочет больше деталей о том, что происходит внутри черной дыры с точки зрения «дождя», я рекомендую почитать про координаты Эддингтона-Финкельштейна:



Они интересны тем, что просто водя по ним пальцем, можно графически дать ответы на много вопросов. В частности, про наши мифы:

  • Свободно падающий наблюдатель достигает горизонта, а затем и сингулярности очень быстро (он падает почти со скоростью света). Если, не долетая горизонта, он передумает и даст «полный назад», то его раздавит в теории он очень долго будет выбираться назад, и вот тогда то он и схлопочет замедление времени по полной.
  • Падающий наблюдатель видит оставленную им вселенную немного замедленной во времени и покрасневшей. Он продолжает ее видеть даже внутри горизонта (свет падает за ним)
  • Падающие наблюдатели так и висят где-то недалеко от горизонта — но это чисто оптический эффект, свету от них очень сложно выбраться наружу
  • Можно бросится им вслед, но как только вы станете приближаться к горизонту, вы увидите что их там уже нет. Если около горизонта висит «подвешенный» наблюдатель, то он сообщит (очень сильно замедленным голосом), что ваш коллега давно уже провалился внутрь. Давно — по данным этого наблюдателя с учетом того что он умный и корректирует свои наблюдения и в курсе того, что он сильно замедлен во времени. По его часам это время может быть очень коротким
  • Более того, они так никогда и не упадут в черную дыру, ведь для этого нужно бесконечное время — ну, вы поняли что это совсем не так

Приливные силы разрывают всякого еще на подлете к черной дыре




Прочность нашего тела такова, что наше тело начнет разрывать примерно за секунду по времени до сингулярности. Учитывая, что свободное падение в черную дыру происходит почти всегда со скоростью, близкой к скорости света, то это произойдет примерно за 300000км до горизонта событий. Учитывая, что обычная черная дыра, являющаяся остатком звезды, имеет радиус 10-15-20км, то нас разорвет задолго до подлета к горизонту.

Однако, есть сверхмассивные черные дыры, массой в миллиарды солнечных масс, а радиусом больше орбиты Урана. В таких черных дырах лететь от горизонта к сингулярности — часы! именно в такие черные дыры можно провалиться, не заметив этого.

Чтобы получить черную дыру, надо сжать материю до чудовищной плотности


Опять таки, это верно для маленьких черных дыр. Радиус горизонта событий пропорционален массе. И это довольно удивительно, ведь при постоянной плотности масса объекта пропорциональна кубу радиуса.

Гравитационный радиус Земли чуть меньше сантиметра. Увеличим радиус Земли в 10 раз (оставив плотность материи такой же). Такая суперземля будет в 1000 раз тяжелее. Гравитационный радиус, соответственно, тоже увеличится в 1000 раз, а объем внутри гравитационного радиуса увеличится в миллиард! раз. То есть в 1000 раз больше материи нам надо запихать в миллиард раз больший объем, то есть теперь сжимать материю надо в миллион раз меньше.

Таким образом, увеличивая масштаб любого тела, мы всегда дойдем до состояния, когда гравитационный радиус «догонит» настоящий. Так что черную дыру можно сделать из любого материала, не сжимая его — из воды, ваты, газа. Даже из вселенной с ее ничтожной средней плотностью — гравитационный радиус вселенной при ее плотности около 10 миллиардов световых лет.

Кстати, это одна из причин, почему невозможны статические решения Вселенной — она бы просто стала черной дырой. Нашей вселенной это не грозит, так как она расширяется.

Есть еще много мифов, если статья понравится, я продолжу.

Комментарии (392)


  1. Nuwen
    03.03.2019 18:04

    «Когда видимая Вселенная была не больше размера атома...»
    А разве это окончательная версия — что Вселенная в самом начале состояла из бесконечного пространства, заполненного «зародышами» видимых вселенных, которые начали раздуваться и разлетаться, в том числе и с бесконечной скоростью относительно друг друга? Ведь считается, что пространство тоже появилось в результате БВ, в чём же находилась первородная бесконечная Вселенная? А ведь ещё и время почти равноценно пространству, и тоже появилось тогда же. У меня то появилась картинка, что вся Вселенная, а не только наблюдаемая, действительно появилась из точки, только эта точка находилась не просто в пространстве, а в 4D пространстве-времени. По 3D аналогии — Вселенная это как расширяющийся из центра шар, где радиусом выступает временное измерение. А поверхность шара — сфера, это наше современное время. Тогда выглядит более органичным выражение, что и материя, и энергия, и пространство со временем, и вообще всё, появилось из центра шара, и начало расширяться во всех направлениях, в том числе и временном. Поэтому мы и наблюдаем реликтовое излучение со всех сторон, ведь оно, по сути, поступает из центра шара. И поэтому же бессмысленнен вопрос — где у Вселенной в пространстве тот центр, откуда она начала расширяться? Центр сферы не лежит на её поверхности, он в центре шара, а по нашему — просто остался в прошлом.


    1. Tzimie Автор
      03.03.2019 18:11

      Рождение baby-universes из «bulk» — это отдельная гипотеза.

      >в чём же находилась первородная бесконечная Вселенная?
      Такой тип мышления череват ошибками, так как исходит от «здравого смысла» — типа, где то это должно было находиться… Здравый смысл — плохой помошних в таких теориях.
      Однако, что касается моделей, которые пытаются объяснить сам big bang, то я знаю как минимум пять, но они дают разные ответы на ваш вопрос.

      >действительно появилась из точки, только эта точка находилась не просто в пространстве, а в 4D пространстве-времени
      Это возможно только для замкнутой вселенной. Мы не знаем, замкнутая наша вселенная или нет (скорее всего нет)

      >И поэтому же бессмысленнен вопрос — где у Вселенной в пространстве тот центр, откуда она начала расширяться?
      Даже для замкнутой вселенной этот центр находится не в пространстве, а в прошлом. Так что указать направление на центр (в 3D пространстве) невозможно

      Впрочем, в одной из моделей с bulk, как ни странно, есть центр у каждой baby universe. Однако это может нас слишком сильно увести в сторону


      1. Nuwen
        03.03.2019 18:31

        Здравый смысл — плохой помошних в таких теориях
        При создании — да, ведь человек создаёт абсолютно новую информацию, и бывает вредно мыслить старыми категориями. Зато он хороший помощник при изучении таких теорий, просто нужен не бытовой здравый смысл, а скорректированный, чтобы теория переставала выглядеть контринтуитивно.
        Так что указать направление на центр (в 3D пространстве) невозможно
        Ну да, я и говорю, так же как и двумерный житель сферы не сможет указать на центр шара.


        1. ebragim
          03.03.2019 19:53
          +2

          Интуиция — в целом бытовой конструкт, который не применим к подобным теориям — потому что основывается на субъективном опыте, не имеющем общего к процессам, происходящим в момент большого взрыва.
          Так же как и здравый смысл.


          1. Nuwen
            04.03.2019 06:39

            Бытовая интуиция. И бытовой здравый смысл. А вот, например, физик Майкл Фарадей развил у себя такую интуицию, которая фактически позволила ему, без применения математического аппарата, основать теорию электромагнитного поля. И не только. Математическим обоснованием занялся потом Максвелл, а Фарадей изобрёл электродвигатель и трансформатор, пользуясь не соответствующими формулами, которых ещё не было, а интуитивно визуализируя потоки электронов и магнитные силовые линии. Вряд ли его субъективный опыт имел до этого что-то общее с процессами электромагнитной индукции.


      1. babylon
        04.03.2019 15:27

        Как может быть бесконечная вселенная если скорость света, а значит и Большого взрыва ограничена? И где-то мы имеем эту границу. Но это при условии, что вселенная — контейнер уже была создана другим Большим взрывом.


        1. Tzimie Автор
          04.03.2019 15:29
          +1

          Вот прочитайте статью)
          Большой взрыв не произошел в какой то определенной точке, а сразу везде. Пожтому ему не надо было куда то там расространяться со скоростью света. И границы между взрывом и невзрывом никакой нет. Опять эта жуткая аналогия со взрывом…


        1. Tyusha
          04.03.2019 16:13
          +2

          Ограничена скорость движения тел (частиц, света) относительно друг друга, когда они прилетают рядом друг с другом (это важно). Скорость растягивания пространства ничем не ограничена. При этом если в двух разных точках такого пространства покоятся два тела, то они могут удаляться друг от друга со скоростью сколь угодно превышающей скорость света, именно потому, что они не в одной точке, а находятся на расстоянии.

          Это мы и наблюдаем во Вселенной. Где-то там существуют галактики, которые удаляются от нас со скоростью больше света. Именно поэтому мы их не видим, они удаляются слишком быстро, что свет от них никогда не сможет достичь нас. Эти галактики находятся за нашим космологическим горизонтом событий.


          1. arthur_veber
            05.03.2019 17:06

            Именно поэтому мы их не видим, они удаляются слишком быстро, что свет от них никогда не сможет достичь нас.

            Если это правда, то как-то это даже обречённо-грустно. В этом есть какая-то тщетность «покорения» космоса.


    1. IsyanovDV
      04.03.2019 04:09

      Есть разные теории, например что вселенная вывалилась из другого-пространства времени, и реликтовое излучение есть остаточное колебание этого явления.
      В любом случае, нет той точки откуда она «взорвалась» (как например взрывается некий заряд в нашем понимании), под взрывом скорее понимается прорыв нашего пространства-времени с образованием элементарных частиц, потому потом собрались в облака, образовали первое поколение звезд, а из остатков первого поколения звезд уже образовались планеты.

      Есть теория, что исходя из 10-12 мерности пространства (теория струн) то, что мы в своем мире видим как материю (фермионы) в других измерениях существует как энергия, оттуда и странные квантовые эффекты в нашем измерении.

      В общем теорий куча, но это теории, проследить историю Вселенной до моменты Большого Взрыва невозможно.


      1. TheShock
        04.03.2019 05:04

        В общем теорий куча, но это теории, проследить историю Вселенной до моменты Большого Взрыва невозможно.
        Я правильно понимаю, что невозможно на наших современных технологиях? То есть, к примеру, если мы каким-то образом сможем взять информацию из других 6-8 измерений, то, возможно, там информация будет больше?


        1. IsyanovDV
          04.03.2019 06:34

          Если теория верна, то возможно в других измерения информации о возникновении нашей вселенной будет больше.

          Но согласно тем же теоретическим выкладкам, эти пространства для нашего мира чрезвычайно «маленькие», и чтобы проникнуть в эти пространства нужно невероятное количество энергии, кто-то из ученых говорил про «коллайдер величиной с Галактику».


        1. Victor_koly
          04.03.2019 09:44

          Проследить в момент БВ не выходит, т.к. практически вся информация ограничена реликтовым излучением (эпоха рекомбинации, около 380 тысяч лет после БВ).
          Есть попытки заметить сам факт инфляции (кроме теории о том, что начальные флуктуации плотности материи вызваны квантовыми флуктуациями в «инфлатонном поле»). Но факт наличия такого поля тоже может оказаться погрешностью наблюдений.


      1. NumLock
        04.03.2019 16:33

        вселенная вывалилась из другого-пространства времени

        Идея заманчивая, но для доказательства этого нужны «железные» мотивы произошедшего. Что могло повлиять на свёртывание (10-12)и-мерного в 3х-мерное пространство. Повышение энтропии? Отбор энергии и упорядочивание подобно механизму кристаллизации? или креационизм? Неизвестно.


        1. TheShock
          04.03.2019 17:38

          Что могло повлиять на свёртывание (10-12)и-мерного в 3х-мерное пространство

          И может ли наше 3-мерное так же свернуться в 2-мерное?


          1. Kriminalist
            04.03.2019 18:56

            Конечно. Как-то так

            Заголовок спойлера
            Корабль полз назад к двумерному участку космоса — сначала медленно, затем постепенно ускоряясь. Двигатели «Омеги» всё так же давали «полный вперед». Это чуть притормозит падение корабля и отсрочит неизбежное, но очень ненадолго.

            В двух тысячах километров отсюда сияние, испускаемое сплющившимся катером и его экипажем, погасло. В отличие от коллапса четырехмерного пространства в трехмерное, переход трех измерений в два сопровождался намного меньшим высвобождением энергии. Две плоские фигуры четко виднелись в звездном свете. На сплющившемся катере просматривались как детали конструкции — кабина, термоядерный реактор и прочее — так и развернувшаяся в плоскость фигура наблюдателя в кабине. В плоском теле другого астронавта четко просматривались кости, кровеносные сосуды и другие органы.

            В процессе выпадания в два измерения каждая точка трехмерного объекта проецировалась на плоскость в прямом соответствии с геометрическими законами, и потому эти две фигуры оказались наиболее полными и точными отображениями первоначально трехмерных катера и людей. Все внутренние структуры лежали теперь бок о бок в двух измерениях, ни одна деталь не перекрывала другую. Процесс проецирования, однако, сильно отличался от того, что используется в технических чертежах, поэтому визуально восстановить первоначальную трехмерную структуру было трудно.

            Самое большое отличие от чертежа заключалось в том, что двумерное развертывание произошло на всех уровнях: все исходные трехмерные структуры и детали легли параллельно в двух измерениях, и результат до некоторой степени повторял эффект, получавшийся при просмотре трехмерного мира из четырехмерного пространства. Это очень напоминало изображения фракталов: сколько ни увеличивай часть изображения, менее сложным оно не станет. Однако фракталы — это теоретические концепции, потому что их действительные изображения неизбежно ограничиваются разрешением: после увеличения в несколько раз изображения теряют свою фрактальную природу. С другой стороны, сложность трехмерных объектов, развернутых в двух измерениях, была реальной: разрешение их лежало на уровне элементарных частиц. Наблюдательный терминал «Омеги» имел ограниченное разрешение, но от сложности и количества подробностей у зрителей голова шла кругом. Это был самый сложный образ Вселенной; если непрерывно смотреть на него слишком долго, можно потерять рассудок.

            И катер, и его экипаж больше не имели толщины.


            1. TheShock
              04.03.2019 19:08
              +1

              Мне кажется, что двумерному миру придется создавать свою жизнь, наша никак не подойдет.


              1. SadOcean
                05.03.2019 11:42

                Так а там не говорится, что наша подойдет.
                Они, очевидно, погибли.
                Просто отпечаток остался.


                1. TheShock
                  05.03.2019 13:25

                  Кстати, в сериале «Орвилл» (который больше похож Стар Трек, чем сам новый стартрек) — в одной из серий они как раз попали в двухмерный мир.


        1. Victor_koly
          04.03.2019 17:57

          Если быть совсем точным, то 10-12 мерного в 4х-мерное. Время в эти измерения точно входит и по идее обособленно. Хотя для НФ про путешествия во времени можно ввести «2-мерное время».


        1. stanislavkulikov
          04.03.2019 18:23

          Ну подождите, разве сингулярность в ЧД в нашем 4х-мерном пространстве — это не 3-х мерное пространство со свёрнутым временем, где место времени занимает радиус? Т.е. я всегда считал, что наша вселенная — это ЧД в 5-мерной вселенной и так далее.


          1. thedeemon
            04.03.2019 18:49

            Обмен ролями между радиусом и временем, в силу изменения их знаков в метрике, это свойство шварцшильдовских координат при применении их к пространству времени внутри ЧД — все, что под горизонтом. Там число измерений то же самое, что снаружи. Более того, это именно свойство координат, в других координатах такого изменения не происходит. Сингулярность же это просто точка или, в случае вращающейся ЧД, тонкое кольцо, там ряд параметров в наших формулах обращается в бесконечность, что тоже больше говорит про наши формулы, чем про природу. Но путать сингулярность со всем объемом ЧД не стоит.


            1. stanislavkulikov
              04.03.2019 19:16

              Ну это в общей теории относительности сингулярность — просто точка. Однако в теории гравитации Эйнштейна — Картана гравитационный коллапс образует так называемый мост Эйнштейна — Розена. Материя достигает огромной, но конечной плотности и «отскакивает», образуя другую сторону моста Эйнштейна — Розена, которая растёт в качестве новой вселенной. И вот размерность этой новой вселенной как раз и будет на единицу меньше.
              Эта теория мне нравится тем, что увязывает все парадоксы: и то, что радиус Хаббла наблюдаемой Вселенной равняется её радиусу Шварцшильда. И свёрнутые измерения. И объясняет тёмную энергию.


        1. IsyanovDV
          05.03.2019 00:40

          У нас нет инструментов чтобы даже предположительно понять, что было до БВ. После БВ остались следы, и развитие Вселенной более-менее не противоречиво описано математически, а вот что было «до» — полная загадка.

          Что касается 10 мерного пространства и вообще теории струн. Понятно, что все это теории, проверить это экспериментально пока нет возможности, но на сегодняшний день «теория струн» — единственная теория, которая не противоречит ни ОТО ни СТО, объединяет все 4 взаимодействия, то есть претендует на место «теории всего».


    1. maxzhurkin
      04.03.2019 20:40

      Если вы планируете дожить до «окончательной версии», у меня для вас плохие новости…


  1. hurtavy
    03.03.2019 18:04
    +2

    Про приливные силы сами ерунду написали. Откуда взялось 1 секунда и 300000 км? Например, для ЧД в 10 солнечных человека порвёт на расстоянии порядка 1000 км. При этом радиус её будет около 30 км.


    1. Tzimie Автор
      03.03.2019 18:15

      Мне врезалось в память с physicsforums. Похоже вы правы, и это расстояние значительно меньше


    1. Victor_koly
      03.03.2019 19:08

      Я не знаю модуль Юнга человека, но можете найти для стального прута.
      ЧД в 10 солнечных масс будет иметь уск. свободного падения 2.74 км/с2 на расстоянии около 700 тысяч км, на расстоянии 70 тысяч вырастет до 2.74*105м/с2.
      То есть факт способности разорвать падающий объект тут важен в сравнении с нейтронной звездой (уникальной будет НЗ массой 2 массы Солнца) или обычным голубым гигантом (он имеет слишком большой радиус при массе 10 масс Солнца и выше).


      1. Tzimie Автор
        03.03.2019 19:11

        >Я не знаю модуль Юнга человека
        Ищу добровольца для эксперимента)


        1. happy-cat
          04.03.2019 13:21

          Интересно это больно? :)
          Не, я серьезно — человек вообще успеет понять что ему больно или даже не поймет что его уже нет и он в Вечности


          1. vsb
            04.03.2019 16:09

            Вряд ли поймёт. Сигналы в нервах и нейронах распространяются довольно медленно. Если ткнуть иголкой, то человек почувствует боль только через доли секунды (отдёрнет руку чуть раньше, но это потому, что рефлекс срабатывает не от мозга). За долю секунды вероятно мозг уже сам будет разрушен вместе с остальным телом.


      1. hurtavy
        03.03.2019 19:45
        +2

        Речь не про ускорение свободного падения, а про его градиент. То есть надо посчитать, насколько у вас ноги будут разгоняться быстрее головы. Так вот, если я правильно прикинул, для ЧД в 10 солнечных на расстоянии в 1000 км приливная сила составит около 350 Н. Это аналогично, если вас подвесить за руки и к ногам привязать груз 35кг. Согласитесь, что это точно не разорвёт


        1. MedicusAmicus
          03.03.2019 23:00

          Если учесть, что пространство тоже искажено, то собственно линейная деформация человеческого тела (если привязать нулевую точку отсчета к голове или ногам) остается под вопросом. А учитывая свободное падение, я думаю, что испытатель будет испытывать натуральную невесомость до момента замедления (столкновения с гипотетической поверхностью).
          И вот тогда — пшик. Очень большой пшик.


          1. dubakov
            04.03.2019 11:46

            Почему невесомость, он же будет ускорено двигаться? То есть двигаясь, например ногами вперёд, ему тяжело будет опустить руки. А если головой вперёд, то поднять.


            1. Tzimie Автор
              04.03.2019 11:52

              Нет. В падающих самолетах создают самую настоящую невесомость


              1. dubakov
                04.03.2019 12:41

                В самолете ускорение константа при невесомости, а при падении в дыру нет.


                1. Daemonis
                  04.03.2019 13:09

                  Нет никакой разницы, падать в дыру или на Землю. Ускорение растет и там и там, просто в случае Земли рост слишком мал, чтоб его заметить.


                  1. dubakov
                    04.03.2019 13:59

                    Именно потому что ускорение ускорения, при падении на землю мало его не заметно. А разница в том, что при падении на землю вас не порвёт ещё до момента касания земли. А при падении на дыру малой массы порвёт именно из-за ускорения ускорения.


                  1. trapwalker
                    04.03.2019 15:51

                    Постойте. Мы же можем «падать» в ЧД так, чтобы оставаться при этом на её орбите, верно? Мне кажется нужно было бы сделать замечание о таком специальном значении слова «падать» в этом контексте. То есть, находясь на некотрой орбите вокруг ЧД наш космческий аппарат будет испытывать приливное воздейтсвие, скорее весего он окажется в приливном захвате, а человек на борту будет комфортно испытвать невесомость пока его тело находится вдоль поверхности гравитирующей массы. Однако развернувшись поперёк он испытает разницу в ускорениях между ногами и головой и будет растянут за означенные части тела с некоторой силой.


                    1. Tzimie Автор
                      04.03.2019 17:13
                      +1

                      Это возможно только далеко от ЧД
                      Ближе к ЧД, ниже 'орбиты света', невозможны никакие замкнутые орбиты для пассивных тел. Однако, вы можете вращаться вокруг ЧД, если использовать двигатели


        1. hoobastank
          04.03.2019 13:15

          А еще можно падать в горизонтальном положении.


          1. Victor_koly
            04.03.2019 13:51

            Можно легко показать, что скажем равнобедренный треугольник момент силы тяжести разворачивает основанием вниз, если речь не про движение по круговой орбите с первой космической.
            В случае с круговой момент центробежной силы наоборот, развернет основание в ту сторону, где выше «потенциал центробежной силы. Именно такая особенность обоих сил приводит к проблеме стпутников. Главная затрата топлива в них не на торможение (если говорить о геостационарной), а на то, чтобы каждые сутки совершить 1 оборот вокруг оси.


        1. trapwalker
          04.03.2019 15:46

          Сударь, мне кажется в данном случае правильнее привязывать человека за ноги и крутить. Вообще, как полагаете правильнее падать в ЧД, головой или ногами вперёд?


          1. maxzhurkin
            04.03.2019 20:46
            +1

            Разница невелика и инструментально её установить практически невозможно


      1. Stas911
        03.03.2019 21:00
        -1

        Но и без Юнга можно оценить, что когда ускорение свободного падения будет больше 20 земных наблюдателю уже сильно поплохеет.


        1. Zolg
          04.03.2019 06:22

          Какая ему печаль до ускорения с которым он свободно падает ?


          1. Stas911
            04.03.2019 06:48

            Ему нет дела до скорости, а до ускорения ему как раз-таки дело есть


            1. Zolg
              04.03.2019 10:26

              Не расскажете, какое именно?

              Какая разница телу, свободно ускоряющемуся в [условно] однородном гравитационном поле, ускоряется оно 10 м/с? или 200?


            1. HappyLynx
              04.03.2019 11:04
              +1

              Дело в том, что свободное падение в гравитационном поле, это не совсем ускорение. По ОТО вы не ускоряетесь, а движетесь по геодезической, т.е. равномерно прямолинейно, но в искревленном пространстве-времени. А разорвет вас из-за того, что геодезические, по которым движутся ваши ноги и голова, в этом самом пространстве-времени расходятся, и как раз вторая производная от функции расстояния между геодезическими в трех пространственных измерениях по функции расстояния между ними же во времени и будет тем самым «ускорением», которое вас разорвет. Причем рассчет влоб даст лишь приблизительную оценку, т.к. модуль Юнга тоже будет зависеть от постоянно изменяющегося градиента координат точек вашего тела по оси времени.

              Как результат, точный рассчет момента разрыва можно давать как очень годную тему курсовой работы студентам физ. факультета.


              1. Zolg
                04.03.2019 12:06

                геодезические, по которым движутся ваши ноги и голова, в этом самом пространстве-времени расходятся
                Что по рабоче-крестьянски звучит как 'гравитационное поле неоднородно'. Эта история отлична от
                когда ускорение свободного падения будет больше 20 земных наблюдателю уже сильно поплохеет
                хуже того, при (средней) напряженности гравитационного поля в 20g получить градиент, опасный для механической прочности тушки, довольно затруднительно


                1. Victor_koly
                  04.03.2019 13:57

                  Со вторым согласен. А по поводу первого — все атомы Вашего тела движутся по какой-то геодезической, просто в уравнении геодезической ещё нужно учесть «все прочие силы» в уравнении, в котором фигурирует 4-ускорение и 4-скорость такого вида:
                  Картинка.


              1. dubakov
                04.03.2019 12:12

                Наверно, ещё нужно учесть скорость передачи гравитационного воздействия от ног к голове


            1. serjmd
              04.03.2019 11:10

              давайте привяжем вам к ногам гирю и выкинем из самолета.


          1. Sergey_Cheban
            04.03.2019 11:40

            Наблюдатель не точечный. Если ноги падают быстрее головы, это уже печально.


            1. Zolg
              04.03.2019 11:53

              'ноги быстрее головы' это про [существенно] неоднородное гравитационное поле.
              исходный же посыл был, что 20g достаточно самих по себе


    1. Kanumowa
      03.03.2019 20:19

      Tzimie по последним данным вроде скорость вращения чд около половины скорости света nplus1, может ли материя на горизонте событий разгоняться до световых скоростей?


      1. Tzimie Автор
        03.03.2019 20:31
        +1

        Момент вращения ЧД может быть разным, зависит от исходного момента коллапсирующего тела. Есть некий «максимальный» момент, который дыра может поглотить. Такая дыра называется «критической». Если черная дыра поглотит еще больше момента, то непонятно что будет (проблемы с теорией и оголяется сингулярность). Считается что большинство реальных черных дыр близки к критическим.


        1. pda0
          03.03.2019 21:48

          Честно говоря не очень понятно в чём именно там проблема. В сверхкритической чёрной дыре материя, чтобы она в такой форме не представляла окажется над горизонтом, т.е. её линейная скорость будет выше скорости убегания. Следовательно она должна покинуть окрестности чёрной дыры. Или проще говоря — чёрную дыру разорвёт центробежной силой.


          1. Tzimie Автор
            03.03.2019 21:55

            в сверхкритической черной дыре нет горизонта.
            по этой причине да, голая сингулярность видна и может даже излучать
            я бы не рекомендовал думать о таких критических объектах в сугубо классических терминах «центробежной силы».
            Чисто интуитивно голые сингулярности мне не кажутся страшными объектами, их существование было бы интересным.

            Компьютерные симуляции дают противоречивый результат: у некоторых получалось такие объекты формировать, у других исследователей критическая черная дыра отказывалась принимать дополнительный момент вращения, буквально отплевываясь от вращающихся тел, которые могли сделать ее сверхкритической.


            1. Tyusha
              03.03.2019 22:17

              Вот интересно, если ей туда не орбитальный момент напихать, от которого, я вполне допускаю, что можно отплеваться, а спин. :) Жаль, что без квантовой гравитации этого не понять.


              1. Tzimie Автор
                03.03.2019 22:26

                именно, непонятно как отплевываться от поляризованных фотонов или электронов, которые туда впрыскиваются с полюсов


                1. Victor_koly
                  03.03.2019 23:45

                  А с другой точки зрения наоборот должен быть такой эффект. Любая обычная звезда для коллапса в НЗ или ЧД должна куда-то скинут спин. Либо он уйдет с внешними оболочками, либо будет выкинут фотонами и гравитонами. Конечно гравитон пока штука гипотетическая, но на Вики (ага, самый авторитетный и свежий источник) написано, что любая теория поля со спином 2 дает физические проявления, как и ОТО.
                  Так что должна выйти разница между потоком вылетающих из ЧД (при формировании, слиянии и т.д.) фотонов и гравитонов с проекциями спина «вверх» и «вниз» относительно общей оси, заданной начальным суммарным вектором орбитального момента вещества.
                  Но так как «с другой стороны от источника ГВ» у нас не своего LIGO, то вроде как проверить этот вопрос не выйдет.


                1. Wizard_of_light
                  04.03.2019 10:27
                  -1

                  Там надо посчитать, не вырастет ли мощность хокинговского излучения с такой дыры. А то может она просто начнёт сильнее светится по экватору, и орбитальный момент будет уноситься излучением.


      1. Tzimie Автор
        03.03.2019 20:32

        Ответ на ваш вопрос ДА — но это относительно далеких, невращающихся объектов.


  1. Dmitry88
    03.03.2019 18:24
    +2

    «Чтобы закрыть тему с Интерстеллар, по современным научным данным, библиотеки внутри черной дыры тоже нет.»
    По фильму ее люди будущего смастерили, как я понял


    1. trapwalker
      04.03.2019 16:03

      Вот отличная отмазка. Как кто-то сказал выше — «на ней заклятье». Ага.


  1. Nuwen
    03.03.2019 18:25

    Свободно падающий наблюдатель достигает горизонта, а затем и сингулярности очень быстро (он падает почти со скоростью света).
    А почему «почти»? Разве что-то помешает ему набрать скорость выше световой — относительно удалённого неподвижного наблюдателя — после пересечения горизонта событий? При ускорении массивного тела, масса этого тела растёт при приближении к околосветовым скоростям, что и препятствует нормальному ускорению, посредством реактивной тяги, например. А при ускорении посредством гравитационного поля, растущая масса уже не помешает телу ускоряться, даже поможет. Кроме того, мне всегда казалось, что первая космическая скорость равна той скорости, которой достигнет падающее тело — ну чисто по законам перехода потенциальной энергии в кинетическую — ведь если закинуть камень в небо со почти первой космической скоростью, то он упадёт обратно по параболе, набрав в конце пути ту же самую скорость, с которой его в начале кинули. То есть, тело, падающее на ЧД, должно, в зависимости от начальной скорости, набрать световую скорость примерно в районе горизонта событий.
    Можно бросится им вслед, но как только вы станете приближаться к горизонту, вы увидите что их там уже нет. Если около горизонта висит «подвешенный» наблюдатель, то он сообщит (очень сильно замедленным голосом), что вам коллега давно уже провалился внутрь
    . Конечно вы такое будете наблюдать, ведь вы в гравитационном поле ЧД, да ещё и движетесь почти со скоростью света. Для вас, и для висящего наблюдателя
    время будет очень замедленно, и с вашей точки зрения, всё падающее на ЧД, уже успеет улететь за горизонт. В то время как остальная Вселенная уже давным давно разлетится на фотоны.


    1. Tzimie Автор
      03.03.2019 18:53

      Пойдем по пунктам.
      >При ускорении массивного тела, масса этого тела растёт при приближении к околосветовым скоростям
      Это миф (ныне не используемое понятие *релятивисткой массы*) — пока не спорьте, я обещаю рассказать про это в выпуске 2

      >что и препятствует нормальному ускорению, посредством реактивной тяги, например
      В СТО тело не может двигаться быстрее света (локально относительно близких объектов), и никакая сила не может ускорить тело быстрее света
      В ОТО сверсветовые движения возможны, но относительно далеких объектов


      1. Victor_koly
        03.03.2019 19:14

        По честному нужно записать уравнение геодезической для 4-скорости пробного тела в скажем метрике Шварцшильда.
        Да, в ОТО (ЕМНИП) нарушится обычное правило
        uiui = 1


    1. Tzimie Автор
      03.03.2019 18:56

      >В то время как остальная Вселенная уже давным давно разлетится на фотоны.
      Как я уже говорил и показано рассчетами, падая в черную дыру внешний мир вы будете наблюдать ЗАМЕДЛЕННО а не УСКОРЕНО

      Что касается «В то время как» вы здесь попали в ловушку «здравого смысла», мы считате что есть некоре общее глобальное время, и что может иметь смысл например такой вопрос: когда я достиг сингулярности, что произошло на земле?" Такой вопрос не имеет смысла. Но имеет смысл другой: пусть с наблюдаю за часами на земле. Какой будет последний отсчет, который я увижу перед сингулярностью? Как уже говорилось, на земле пройдет немного


      1. Nuwen
        03.03.2019 19:12

        внешний мир вы будете наблюдать ЗАМЕДЛЕННО а не УСКОРЕНО
        Наблюдать. Так же как вы говорите про зависшие на горизонте объекты, что это не более чем иллюзия.
        считате что есть некоре общее глобальное время
        Я понимаю, в чём заморочка с общим временем, я говорил про время остальной Вселенной, а не про общее время.
        Ладно, я подожду второй части и пока порисую графики.


      1. saboteur_kiev
        04.03.2019 05:42

        Вообще-то, у меня есть подозрение, что «замедление времени» конечно более сложный процесс, но можно упростить его понимание до банальной механики.

        Если гравитационные силы настолько велики, что вмешиваются в микропроцессы замедляя движение молекул, атомов и частиц, то наблюдать там уже ничего не выйдет. Ведь время — это по сути скорость протекания процессов.

        Ни техника ни живое существо существовать в таких условиях не сможет. И подозреваю, что это случится задолго то семикратного замедления. Поэтому даже мысленный эксперимент про «наблюдателя» некорректен.


        1. Bronx
          04.03.2019 06:57

          Если замедляется абсолютно всё, даже скорость света, то ни техника, ни живые существа этого замедления никак заметить не смогут.


          1. saboteur_kiev
            04.03.2019 16:58

            В том-то и дело, что не может все «просто замедляться».
            На микроуровне происходят процессы, которые влияют на скорость прохождения реакций на более высоких уровнях.

            Например, вы говорите замедляется скорость света. Это косвенно означает, что замедляется движение всех частиц, включая, например, электроны?
            Что происходит с атомами, на орбите которых летали эти электроны? Атомы еще остаются атомами или структура веществе необратимо меняется?


            1. vedenin1980
              04.03.2019 17:05
              +1

              По ОТО ничего не происходит, так как пространство так же искажается. Более того, замедление времени относительно — для пролетающего мимо Земли звездолета, время замедлиться на Земле, а на Земле — наборот. Поэтому не может замедляться, что-то одно иначе мы все бы развалились на части. Вас же не удивляет, что относительно релитового излучения мы летим с бешеными скоростями, но при этом сами этого никак не замечаем. Точно так же с временем, с точки зрения локального наблюдателя течение времени остается совершенно обычным, это скорость времи всех остальной Вселенной становится другой.


              1. saboteur_kiev
                04.03.2019 17:42
                -2

                Более того, замедление времени относительно — для пролетающего мимо Земли звездолета, время замедлиться на Земле, а на Земле — наборот.

                То, что в чайнике рядом со стаканом вода вскипает, не означает, что для воды в стакане в кипящей воде ускорилось время и наоборот. IMHO это все же спекуляция терминологией. Просто для живого наблюдателя время критично в силу его ограниченного существования.

                Поэтому не может замедляться, что-то одно иначе мы все бы развалились на части.

                Конечно не может. Но я имел ввиду, что происходящее на макроуровне — следствие того, что происходит на микроуровне.

                Мне не нравится сам термин «Скорость времени». Мне кажется, что более логичным является термин «скорость протекания процессов».


                1. Victor_koly
                  05.03.2019 09:36

                  Но тогда всех процессов. Как процессов с точки зрения диф. уравнений движения, так и с точки зрения всех 3 взаимодействий.
                  P.S. Ещё желающие могут узнать, описывается ли распад бозона Хиггса «скоростью» процессов поля Хиггса.


            1. Bronx
              04.03.2019 22:44

              Фактически, скорость света не может замедляться или ускоряться, потому что её величина условна, это всего лишь соотношение между двумя измерительными линейками: линейкой метров и линейкой секунд. Можно спокойно переделать все линейки, приняв c=1 (планковская система величин) — и вся физика останется неизменной. Релятивистское «замедление времени» эквивалентно повороту в 4-пространстве, которое изменяет видимые пропорции линеек, но никак иначе не влияет на материю и локальные процессы. Атомы остаются атомами, просто со стороны стороннего наблюдателя они двигаются медленнее, а внутренний наблюдатель ничего не замечает. Только если наблюдатель сильно распределённый, разница протекания процессов в разных частях системы начнёт на него ощутимо влиять.

              ЗЫ. Вообще, нужно помнить, что величина любой физической константы, имеющей размерность, конвенциальна, и может быть безболезненно изменена переопределением «метра», «секунды», «килограмма» и т.п. Истинные константы — безразмерны (постоянная тонкой структуры, например).


              1. thedeemon
                05.03.2019 01:35

                В СТО все так. В ОТО еще нужно учитывать эффекты геометрии. Так же, как длина одного градуса долготы различна на экваторе и в Питере, хоть в метрах ее считай, хоть в милях, в ОТО в разных точках пространства могут отличаться временные отрезки и длины, в каких единицах ни считай. Т.е. двое часов, пролетевшие из А в Б одним маршрутом и другим, могут показать разное время, в чем бы они ни измеряли время.


                1. Bronx
                  05.03.2019 05:43

                  Это относится к случаю протяжённого наблюдателя, про который я упомянул отдельно.


        1. Victor_koly
          04.03.2019 09:49
          -1

          Есть у кого-то теория, что замедляется скорость процессов, но только электромагнитных.
          Конечно ни одно живое существо не переживет, если между его мозгом и сердцем скачек («плавный», так сказать) метрики обеспечит разницу течения времени в 137 раз.
          А так, я не знаю, когда разорвет стальной прут возле ЧД массой 3 массы Солнца.


          1. vedenin1980
            04.03.2019 17:11
            +1

            Есть у кого-то теория, что попав в ЧД вы окажетесь в мире, где летающие пони какают радугой. [/sarsacm]

            ОТО вполне четко говорит, что замедлятся все процессы и все эксперименты это подтверждают, более того множество выкладок показывают, что изменение только части процессов из-за относительно скорости — сломает вообще все. Земля ведь тоже движеться со скоростью близкой к световой относительно определеных точек отсчета, то есть даже у нас на Земле мы должны это наблюдать.

            А вы пишите где-то у кого-то есть какая-то теория, ну дайте тогда ссылки на эту теорию, причем желательно с математическими выкладками.


            1. Victor_koly
              05.03.2019 09:53
              -1

              А «альтернативщиков» редко бывают математические выкладки. Такие теории происходят наверное от «эфирщиков», но точно не помню.
              А так понятно, что скажем в СТО замедлялась скорость распада мюонов, а это слабое взаимодействие. Доказательство тому — детектор LHCb в процессах с sqrt(s) = 8 ТэВ фиксировал разпад W-бозона на мюон и антинейтрино (заточен именно под мюоны тригер «запиши событие»).
              Или скажем происходит рождение пары Z-бозонов (тот самый сеанс LHC, но детектор CMS) с распадом на пары «электрон-позитрон» или «мюон-антимюон». Эти 4 частицы рождаются с огромной инвариантной массой в диапазоне от 110 до 1100 ГэВ. Например поперечный импульс пары свыше 45 ГэВ.
              Так как ОТО — общий случай для СТО, она не будет так уж сильно меняться закон, дающий частице с массой покоя m иметь кин. энергию T >> mc2


      1. stanislavkulikov
        04.03.2019 15:14

        Я вот не понимаю, почему находясь на Земле, я наблюдаю немного ускоренную версию вселенной, находясь на Юпитере, я буду наблюдать ещё более ускоренную вселенную. Но вот в ЧД внешний мир вдруг замедлится. Почему?


        1. Tzimie Автор
          04.03.2019 15:35
          +1

          Потому что на Земле вы НАХОДИТЕСЬ а в черную дыру вы ПАДАЕТЕ
          Если вы будете ВИСЕТЬ недалеко над горизонтом, то вселенная будет выглядеть ускоренной


    1. thedeemon
      04.03.2019 15:02
      +3

      >Разве что-то помешает ему набрать скорость выше световой — относительно удалённого неподвижного наблюдателя — после пересечения горизонта событий?

      Тут сразу несколько чисто математических проблем. Скорость это вектор, а в используемой в ОТО римановой геометрии осмысленно сравнивать вектора мы можем лишь в одной точке. Если у нас через одну точку пролетают одновременно два тела, мы можем сравнить их скорости. Если же они далеко друг от друга, то объективного ответа «какова скорость второго тела в системе отсчета первого» уже нет, т.к. результат зависит от того, как мы проводим в пространстве-времени множество одновременных событий, и даже по какому пути мы переносим вектор скорости из одной точки в другую для сравнения векторов.

      Мы можем использовать метрику Шварцшильда в привычном виде, она описывает как все выглдядит для удаленного статичного наблюдателя. Но для такого наблюдателя, в такой системе отсчета, падающий в ЧД объект никогда не пересечет горизонт событий, так и будет вечно к нему приближаться все медленнее. (Считаем ЧД тоже статичной, с испарающейся все сложнее) Получается, что осмысленно говорить о скорости кого-то упавшего под горизонт в такой удаленной системе отсчета не получится. Даже если думать не о том, кто падает в ЧД снаружи, а уже там внутри как-то оказался, в привычных координатах Шварцшильда внутри ЧД знаки у времени и радиуса меняются местами в метрике, радиус начинает работать как время, и внутри ЧД у всех объектов скорость как бы быстрее света (для внешнего наблюдателя), но это не настоящая скорость, а то, что внутри ЧД можно назвать скоростью, для стороннего наблюдателя выглядит не как метры радиуса в секунду времени, а наоборот, секунды времени в метры радиуса, т.к. роли этих измерений поменялись. Но это уже спекуляции, проще и корректнее считать, что все, что под горизонтом, уже за пределами пространства-времени внешнего наблюдателя, так что его координатами там пользоваться смысла нет.


      1. Tzimie Автор
        04.03.2019 15:04

        Как уже говорилось, перейдите из Швардшильдовой метрики в Eddington или Kruskal и проблемы исчезнут. Бесконечность на event horizon чисто техническая, связанная с неудачным выбором метрики.


        1. thedeemon
          04.03.2019 15:41

          Во-первых, это та же самая метрика просто в других координатах.
          Во-вторых, такая смена координат отражает смену «точки зрения», для статичного удаленного наблюдателя шварцшильдские координаты ближе всего к натуральным, координаты Крускала же ближе тому, кто падает.
          В-третьих, проблема выбора общего «сейчас» и сравнения векторов никуда не девается. Есть два корабля у ЧД, какой момент истории одного корабля одновременен текущему моменту истории другого? Ответ не так однозначен же.


          1. Tzimie Автор
            04.03.2019 15:45
            +2

            >Во-первых, это та же самая метрика просто в других координатах.
            Да, вместо «координат» написал «метрика». Но главное, бесконечности там нет

            >Во-вторых, такая смена координат
            Да

            >В-третьих, проблема выбора общего «сейчас» и сравнения векторов никуда не девается
            А в ОТО в общем случае НЕТ глобального «сейчас»


            1. thedeemon
              04.03.2019 16:08
              +3

              Вот об этом и речь. Выше человек спрашивал про скорость относительно внешнего наблюдателя. Ее фиг определишь однозначно.


  1. TheShock
    03.03.2019 18:25
    +2

    Для того, чтобы замедление времени было бы таким, планета должна была быть на расстоянии чуть более 0.003% от радиуса Шварцшильда. Между тем, судя по виду на черную дыру
    Самый популярный миф, на который Кип Торн отвечал в интервью:

    Время там идёт замедленно — один час на ее поверхности равен семи земным годам.

    ПРЕТЕНЗИЯ: Такое возможно только вблизи объектов, обладающих огромной массой, например, на орбите черной дыры. Но нужно находиться совсем рядом с дырой, практически над ее поверхностью. А стабильная орбита вокруг черной дыры должна превышать диаметр Гаргантюа как минимум трижды. Иначе планету Миллер давно бы засосало внутрь. С учетом показанных в фильме кадров время на поверхности планеты должно течь медленнее, чем на Земле, всего процентов на двадцать.

    Это верно в отношении невращающихся черных дыр, но с Гаргантюа все обстоит по-другому. Гаргантюа — сверхмассивная вращающаяся черная дыра, что несколько меняет ее воздействие на окружающее пространство. При определенных условиях, скажем, если она будет вращаться очень быстро, а планета Миллер — располагаться достаточно близко к циркулярной орбите Гаргантюа, такое замедление времени возможно.

    Правда, у вращающихся черных дыр есть предел скорости вращения, причем максимума они, как правило, не достигают. Чтобы на планете Миллер было такое замедление времени, Гаргантюа должна вращаться лишь чуточку меньше максимума. Это реально, хотя и маловероятно.



    1. Tzimie Автор
      03.03.2019 18:47
      +1

      Это отмазка с его стороны
      Такое замедление возможно только у эргосферы, которая находится недалеко от горизонта событий, где то в райное 1.5радиуса в случае максимального вразения черной дыры. И то только при «ретроградном» вращении, что почти невозможно для планеты. Кроме того, те же эффекты увлечения около эргосферы сразу бы разорвали планету при малейшем ее вращении.

      По этой причине я предпочитаю fantasy, сказали бы про что на планету наложено заклятье, и никто бы и не вякал.


      1. DayFan
        05.03.2019 22:03
        +1

        С одной стороны «отмазка» профессора и лауреата нобелевской премии, с другой стороны ваши допущения. Даже не знаю кому верить.


    1. Victor_koly
      03.03.2019 18:58

      Про свойства вращающихся ЧД ничего толком не знаю.
      По поводу планет, на поверхности которой скорость течения времени в разы меньше, чем на орбите. В самом грубом приближении скорость течения времени — это функция потенциала GM/r. Отношение разности потенциалов между 2 точками на линии к центру ЧД к расстоянию между точками — это будет средняя сила гравитации на этом участке. Тут вспомнил, что нужно добавить — если на участке «планета-орбита» средняя гравитация 100 «же», значит на планете она как минимум больше.
      Без фантастической «большой ЧД» (грав радиус хотя бы 0.1 а.е.) мы не получим большой разности течения времени, если только «на орбите вокруг планеты» — это не точка много дальше точки Лагранжа (номера не помню, той что дальше по лучу «звезда-планета»).


  1. catsmile
    03.03.2019 18:27

    Ещё!


    1. Tzimie Автор
      03.03.2019 18:48

      Будет!


      1. igordata
        04.03.2019 17:31
        +2

        И аватарку поставь, чтобы легче узнавать! =)


        1. MMTRIX
          06.03.2019 00:51

          Шелдона — разоблачителя!)


    1. KvanTTT
      04.03.2019 02:44

      Можно еще про нерешенные проблемы современной физики, если, конечно, есть что сказать.


    1. neurocore
      04.03.2019 06:39

      Ещё стартрек претендует на научность, можно оттуда поразбирать, что близко к реальной науке


      1. TheShock
        04.03.2019 08:40

        В последнем сериале Стартреку они мгновенно путешествуют по миру вставляя провода в животное. Меня «порадовало».


        1. neurocore
          04.03.2019 08:42

          Не, ну я конечно имел в виду классику: оригинал, следующее поколение, DS 9


          1. Victor_koly
            04.03.2019 10:05

            В TNG/DS9/VOY уже все весьма далеко от научности. Технический прогресс достиг небывалых высот.
            Просто смотря TOS видеш такой примитивный уровень технологий, придуманный в начале 60х годов для 2270 года. Потом смотришь ENT и удивляешься, почему в 2150 все весьма развитое. А потом вспомнинаешь, что скажем у Стругацких в 2150 уже весьма развитая цивилизация.


          1. TheShock
            04.03.2019 10:52

            Та я так, вспомнил грустную историю.


        1. Kriminalist
          04.03.2019 13:00

          Зачем вы назвали астромиколога лейтенанта Пола Стеймица животным??


          1. TheShock
            04.03.2019 13:17

            А вы видели, как они с тихоходкой обращаются? Настоящие животные!


  1. Space__Elf
    03.03.2019 18:53
    +1

    У меня такой вопрос:
    — есть парадокс, согласно которому, если два корабля связанные тросом синхронно ускоряются до скорости света, то трос разорвёт
    — я правильно, понимаю, что этот эффект при приближении к скорости света разорвёт и сам корабль тоже?


    1. Tzimie Автор
      03.03.2019 19:05

      О, да, паравозик Эйнштейна. О него было сломано много копий.

      Действительно, чтобы длинный жесткий объект ускорить, разные его части должны двигаться с разным ускорением (хвост быстрее). Если ускорение небольшое (реактивный двигатель) то ничего не разорвет, но ускоряться будете долго — жесткость корпуса все компенсирует (корабли немного сожмет что анрушает идеальность ускорения).

      Однако здесь есть более глубокая тема: если объект ОЧЕНЬ ДЛИННЫЙ, то есть максимальное ускорение, которое он может выдержать, оставшись целым. Потому что хвост не может двигаться быстрее скорости света. Поэтому если вы ускоряетесь и обернетесь назад, то где то очень далеко будет область, которая вас никогда не сможет догнать. Там образуется (только для вас!) видимый горизонт (как у черной дыры!) от которого идет излучение Хокинга. От него вам будут лететь фотоны (которые видите только вы!). Это пример того, что в квантовой механике ускоренные наблюдатели не всегда будут согласны в том, существуют какие либо частицы и сколько их.

      Про все это безумие можно начать читать тут: en.wikipedia.org/wiki/Unruh_effect


      1. Space__Elf
        03.03.2019 19:08

        А посчитать на какой скорости при заданном ускорении (к примеру 1g) разорвёт, к примеру километровый корабль, как?


        1. Tzimie Автор
          03.03.2019 19:13
          +2

          Не скорости а ускорении!
          Если ваш реактивный двигатель магическим образом ускоряет вас 1g, то мы можете ускаряться хоть год и чувствовать все тот же 1g. Для других мфы будете укорачиваться и ускоряться все меньше и меньше, приближаясь к скорости света.


          1. Space__Elf
            03.03.2019 19:16

            То есть, при 1g можно ускоряться приближаясь к скорости света, не боясь, того что корабль разорвёт?


            1. Tzimie Автор
              03.03.2019 19:20
              +1

              Конечно.
              Про разрыв струны там другое: ru.wikipedia.org/wiki/%D0%9F%D0%B0%D1%80%D0%B0%D0%B4%D0%BE%D0%BA%D1%81_%D0%91%D0%B5%D0%BB%D0%BB%D0%B0


            1. Tyusha
              03.03.2019 19:33
              +1

              Вы зря прицепились к приближению к скорости света. Скорость относительна, поэтому вы даже сами можете поставить эксперимент и узнать разорвёт ли вас при приближении к скорости света!

              Представьте, что мимо Земли пролетает инопланетный космический корабль с скоростью 0,99с. Для него именно вы движетесь со скорость 0,99с. Теперь вы садитесь в автомобиль, начинаете ускоряться (пусть со ускорением 1g, неважно)… и вас разрывает. Так что ли?


            1. pda0
              03.03.2019 21:56

              Только если вы летите в идеальном вакууме. В реальном космосе энергия столкновений с космическими лучами, свободными атомами и прочим мусором будет расти с вашей скоростью и в какой-то момент вас просто поджарит ими.


      1. Victor_koly
        03.03.2019 19:19
        +1

        Это не меньшее безумие, чем рождение внутри протона пары частиц с массой 8-9 масс покоя протона. Там «ускоренный наблюдатель» не согласен с составом протона будет.


    1. TheShock
      03.03.2019 19:13

      — есть парадокс, согласно которому, если два корабля связанные тросом синхронно ускоряются до скорости света, то трос разорвёт
      Вы уверены, что есть такой парадокс? Разве трос вообще заметит изменения в точке отсчета кораблей?


      1. Tzimie Автор
        03.03.2019 19:17
        +1

        1. TheShock
          03.03.2019 19:40
          +1

          Дамс, все так сложно с этой физикой.


    1. Tyusha
      03.03.2019 19:25

      Нет, не правильно.

      В парадоксе с тросом, как почти во всех парадоксах теории относительности, не принимается в расчёт относительность одновременности в разных системах отсчёта. При одновременном ускорение в сопутствующей системе отсчёта связанных кораблей трос не порвётся.


      1. Tzimie Автор
        03.03.2019 19:29

        Да, ключевое слово — сопутствующей
        А при одновременном ускорении в «земной» — порвется.

        Вообще интересно что даже знаменитый Бэлл известный своей теоремой и не только запутался в этом


        1. Tyusha
          03.03.2019 19:38

          Ну я не вдаюсь в тонкости. Понятно, что имеется ввиду общая инерциальная СО кораблей. Излишне вам говорить, что ускоренная СО не может быть глобально инерциальной, а только локально. Поэтому такой СО нельзя «хорошо» накрыть и передний, и задний корабль, сколько-нибудь удалённые друг от друга.


          1. TheShock
            03.03.2019 19:41

            сколько-нибудь удалённые друг от друга.
            А в цифрах это сколько? Километры? Метры?


            1. Tyusha
              03.03.2019 20:04

              Зависит от ускорения. Из размерности задачи (есть только два параметра ускорение А и скорость света С) можно составить только одну величину размерности длины: С^2/А. Это и будет характерное расстояние. При 1g оно составит примерно 10^16 метров. Это где-то 1 световой год.


              1. Victor_koly
                03.03.2019 23:48

                Чуть меньше 1 светового года, да. Или чуть больше 1 месяца на «12-кратных», но я не помню, читал ли эту книгу Стругацких. Хотя понятно, что если разгоняться за 12 месяцев внешней СО, то на последнем месяце будут огромные перегрузки.


  1. Varim
    03.03.2019 19:02

    Гравитационный радиус Земли чуть меньше сантиметра.
    Разве это так? Ведь в центре земли невесомость, так как вещество в центре земли тянется во все стороны одинаково. Разве там есть такая плотность что бы был Гравитационный радиус? Наверно самая высокая плотность лежит чуть дальше от центра земли. Дальше — может на сантиметры, а может на метры?


    1. Space__Elf
      03.03.2019 19:05

      Гравитационный радиус — это размер до которого нужно сжать тело, чтобы оно приобрело свойства чёрной дыры.


      1. Varim
        03.03.2019 19:10
        +1

        Ага, «Гравитационный радиус Земли чуть меньше сантиметра» — это наверно следовало читать как «если бы землю сплющили до 1го кубического см»


        1. Tzimie Автор
          03.03.2019 19:13

          Да


        1. Victor_koly
          03.03.2019 19:23

          Грав. радиус удобно вводить для любого тела со сферической симметрией, т.к. через него можно записать вокруг тела метрику Шварцшильда. Просто при грав. радиусе 1 см получится, что расстояние 9 км от уровня моря не очень сильно отличается от точки с расстоянием 10^9 км, хотя точные эксперименты с атомными часами дали выявить эффекты ОТО даже на масштабе орбиты высотой несколько десятков тысяч км (требуется учитывать в работе спутников GPS).


  1. mikeus
    03.03.2019 19:21
    +1

    Есть еще много мифов, если статья понравится, я продолжу.
    На самом деле хотелось бы не разбор и критику мифов, а ясный не предвзятый обзор того, что является проверенными на экспериментах моделями, а что — лишь теориями, будоражащими воображение неискушённой публики своей грандиозностью, неизведанностью, космической фундаментальностью.
    Иначе всё это превратится в очередную типичную дискуссию участников собрания свидетелей Иеговы большого взрыва.


    1. Space__Elf
      03.03.2019 19:24

      * не туда написал *


    1. Tzimie Автор
      03.03.2019 19:31

      Здесь я пытаюсь ограничится только научными теориями, неправильно подаваемые Discovery и ище с ним. Про Unruh effect в комментах разве что ступил в область гипотетических построений


    1. dm9
      03.03.2019 20:38

      Из научно-популярного посмотрите на Ютубе лекции Сергея Попова по астрофизике. Он всегда очень аккуратно обращается с такими понятиями как "доказано", "гипотеза", "стандартная гипотеза" и максимально ссылается на публикации. При этом интересно рассказывает.


    1. Miamy
      04.03.2019 00:05

      Простите, а как можно экспериментально проверить модель чёрной дыры или большого взрыва?


      1. Tzimie Автор
        04.03.2019 00:06

        Тут наука выходит на границы возможности применения принципа Falsifiability.
        И это проблема


        1. Shkaff
          04.03.2019 10:05

          Ну почему же? Модели черных дыр можно проверять по косвенным наблюдениям — динамике окружения или излучения, и по прямым — наблюдая гравитационные волны.
          Большой взрыв до какой-то степени можно проверять по реликтовому излучению, балансу частиц и т.п. По крайней мере какие-то классы моделей это поможет отделить.


      1. Victor_koly
        04.03.2019 10:06

        Был эксперимент с «аналогом» излучения Хокинга.


        1. Shkaff
          04.03.2019 10:12

          Ну это как раз никак не поможет проверить модель черной дыры. Вообще, кроме любопытного эффекта никакой пользы в сравнении аналоговых моделей с астрономическими нет.


  1. Space__Elf
    03.03.2019 19:24
    +1

    А что с ненулевой плотностью межзвёздного газа?
    На какой скорости он начнёт оказывать воздействие на корабль?


    1. Tzimie Автор
      03.03.2019 19:33
      +2

      Там все плохо. Там не только газ, там любая пылинка приводит к фатальным последствиям. Собственно, любая пылинка имеет столько энергии, что ее достаточно чтобы расплавить космический корабль. Так что никаких полетов до mind upload.


      1. Space__Elf
        03.03.2019 20:04

        И на какой скорости, такая пылинка будет иметь такую энергию? (как оценочно примерно посчитать и прикинуть?)

        никаких полетов до mind upload

        Почему же? Старый добрый корабль поколений — точно безопасен.


        1. Victor_koly
          03.03.2019 23:50

          Вы классику читали? Где только 1 человек на корабле знал, как подать команду «Приземляемся в этой системе».


        1. a5b
          04.03.2019 08:09

          Пример для пылинок на 0.1c — Иван Александрович Корзников «Реальности межзвездных полетов», 2006 — go2starss.narod.ru/pub/E009_RMP.html (оригинал — zhurnal.lib.ru/k/korznikow_i_a/realxnostimezhzwezdnyhpoletow.shtml)

          > Такие частицы, сильно рассеивающие и поглощающие свет, имеют размеры 0.1-1 микрон и массу порядка 10^-13 г
          > При v=0.1c… Легко найти, что пройдя путь в 1 световой год, экран звездолета встретит n=rs=10^-12 · 9.46 · 10^17 =10^6 пылинок на каждый см^2, и каждые 500 пылинок сроют слой 0.448 мм экрана. Значит, после 1 светового года пути экран будет стерт на толщину 90 см…
          >… не менее 5 частиц массивнее 0.1 г на все поперечное сечение звездолета. А каждая такая частица при v=0.1c имеет энергию более 4.53·10^10 Дж, что эквивалентно кумулятивному взрыву 11 тонн тротила.
          > Поэтому межзвездные перелеты могут осуществляться лишь с существенно меньшими скоростями, порядка 0.01с и менее.

          Немного еще есть в Pros and Cons of relativistic interstellar flight,
          Oleg G. Semyonov, arxiv.org/ftp/arxiv/papers/1807/1807.08608.pdf
          The interaction of relativistic spacecrafts with the interstellar medium, arxiv.org/abs/1608.05284
          и из en.wikipedia.org/wiki/Interstellar_travel#Interstellar_medium

          Карта местности и проекты на 300 лет — marsmeta.narod.ru/univ/unistep.html
          > Заключение. При выбранных условиях: скорость звездолета 0.1·с и время перелета не больше 50 лет человечество не сможет распространить волну цивилизации дальше системы Центавра и звезды Барнарда. Теоретически возможны единичные исследовательские полеты к Сириусу, Лаланаду 21185 и Вольфу 359. Однако колонизация окрестностей этих звезд растягивается на столь большое время, что делает это предприятие сомнительным.


          1. niknamezanat
            04.03.2019 10:03

            /trololomode
            При достижении определённой скорости перестают действовать силы межмолекулярного и межатомного взаимодействия, и тело превращается в набор несущихся невзаимодействующих частиц. При это, локально, оно продолжает «ощущать» себя целым и невредимым. Далее. Поскольку этот набор несущихся частиц явно не будет обладать строго параллельными траекториями у каждой частицы и будет не сфокусирован, то тело будет расширяться и «утончатся». После определённого расстояния, даже планеты и звёзды не доставят каких-либо проблем, т.к. они будут проходить это истончившиеся тело гигантических размеров насквозь, не провзаимодействовав ни разу.


            1. Tzimie Автор
              04.03.2019 11:56
              +1

              Снова Рен ТВ


              1. niknamezanat
                04.03.2019 13:23

                Да, не. Телек не смотрю. У дураков мысли сходятся, просто.
                А если серьёзно, то любое тело испариться уже на этапе разгона, а он возможен только в инерциальной системе.
                Правда, я не знаю, на какой конкретно. Хотелось бы чего-нибудь по этой теме, но поиск не очень помогает.


            1. vlad9486
              04.03.2019 14:34

              Скорость относительна. Скорость может быть только относительно чего-то, само по себе тело не может иметь скорости и что-то там переставать действовать тоже не может.


              1. niknamezanat
                04.03.2019 14:50

                что-то там переставать действовать тоже не может

                может и даже обязано. Ни одно физическое тело невозможно разогнать до около световых скоростей из-за этого в принципе.


                1. vlad9486
                  04.03.2019 15:02
                  +1

                  Разогнать не возможно. Потому что что-бы раньше лететь медленнее, а сейчас быстрее нужно тоже самое тело отсчета относительно которого летим. А так можете считать что вы прямо сейчас летите с околосветовой скоростью.


                  1. niknamezanat
                    04.03.2019 15:52

                    Зачем мне так считать?


                    1. vlad9486
                      04.03.2019 17:01

                      Что-бы свести до абсурда изначальное утверждение

                      При достижении определённой скорости перестают действовать ...


                      1. niknamezanat
                        04.03.2019 21:13

                        Абсурда что-то не наблюдаю.

                        При достижении определённой скорости
                        относительно чего-нибудь, раз так угодно. Не думал, что тут есть такие, для кого это необходимо уточнять.
                        Более того, не понимаю, с чем спорите-то?


                        1. vlad9486
                          05.03.2019 12:35
                          +1

                          У вас там написано

                          При достижении определённой скорости перестают действовать силы межмолекулярного и межатомного взаимодействия
                          а потом вы написали
                          относительно чего-нибудь, раз так угодно
                          А фокус в том что относительно одного тела рассматриваемое тело достигло определенной скорости а относительно какого-то другого тела не достигло. Это означает что относительно одного тела силы межмолекулярного и межатомного взаимодействия перестают действовать, а относительно другого тела не перестают. Понимаете? Скорость может быть разной в разных системах отсчета. Силы же не могут быть разными.


                  1. Welran
                    05.03.2019 14:48

                    Вроде как не просто можете, а все частицы всегда двигаются со скоростью света в четырехмерном пространстве времени.


                    1. vlad9486
                      05.03.2019 14:58

                      Тогда уж лучше так, скорость — это гиперболический тангенс угла между касательными к траекториям в четырехмерном пространстве. Потому скорость может быть только между двумя. Угол не получится если прямая только одна.

                      И процессы между этими двумя телами зависят от скорости, камень на скорости 1 м/с об голову совсем не тоже что на скорости 100 м/с. Но вот процессы в одним и том же теле не могут зависеть от его скорости относительно какого-то произвольного другого.


          1. antihydrogen
            04.03.2019 12:22

            Вот серьезная статья на тему столкновения космических аппаратов с пылинками arxiv.org/pdf/1608.05284v2.pdf

            Согласно ей, при скорости v=0.2c на пути длиной 4 световых года от ударов пылинками графитовый экран потеряет на испарение слой толщиной 0.7 до 2 миллиметров (в зависимости от плотности межзвездной среды в наших окрестностях). Кварцевый экран — от 0.5 до 1.5 миллиметров.

            Из Рис.16 в статье видно, что при скорости v=0.1c потери на испарение для кварцевого щита будут 0.3 мм максимум.

            Иначе говоря, Иван Александрович Корзников в своих оценках ошибся аж на 4 порядка вверх. Причины этой ошибки:
            1) Выбор титана в качестве материала щита в его оценках (довольно нелепый выбор, так как при столкновениях на 0.1с для материала щита важна не прочность, а удельная теплота испарения).
            2) Неверное понимание физики процессов при столкновении с пылинкой и, соответственно, использование для оценок потери материала соотношения, выведенного для скоростей много меньше 0.1с.

            Вывод: для близких межзвездных перелетов проблема столкновения с пылинками не очень существенна. Даже для тонких аппаратов вроде Starchip — их после окончания разгона можно просто повернуть ребром по направлению движения.


    1. zahmTOD
      03.03.2019 19:56

      На любой. Разница только в силе (грубо) воздействия.


  1. Manyak872
    03.03.2019 19:34
    +1

    Для того, чтобы замедление времени было бы таким, планета должна была быть на расстоянии чуть более 0.003% от радиуса Шварцшильда.

    А вот П. Р. Амнуэль, ссылаясь на Кипа Торна (того самого консультанта фильма и по совместительству Лауреата Нобелевской премии по физике 2017 года), в статье Черная дыра в кинематографе, опубликованной в журнале «Химия и жизнь» (№3, 2015), утверждает, что
    эффекта [...] действительно можно добиться, но лишь в случае, если черная дыра вращается так быстро, что ее горизонт событий, то есть то, что можно считать границей дыры, движется со скоростью, которая меньше скорости света всего на 0,01 миллиметра в секунду! Только тогда пространство-время вокруг нее искривляется и закручивается столь странным образом, что и время замедляется в сто раз, с точки зрения удаленного наблюдателя, и гравитационное поле уравновешивается центробежной силой, благодаря чему и тело астронавта, и корпус космического корабля не испытывают чудовищных перегрузок.


    1. Tzimie Автор
      03.03.2019 19:35

      Возможно. Но также очевидно, что эта область КРАЙНЕ узкая. Сотни метров, километры. Планеты точно туда не поместится, и ее разные стороны будут в совершенно разных условиях


      1. Manyak872
        03.03.2019 21:14
        +1

        Ну так и у нас на Земле разные стороны в совершенно разных условиях) Это похоже на противопоставление «запрещено» против «не запрещено, но маловероятно» Торна. В этом конкретном вопросе я бы примкнул к лагерю Торна.


    1. Tyusha
      03.03.2019 19:43
      +3

      Будь то чёрная дыра, или не чёрная, или не дыра вовсе, космонавтам пофиг структура и природа центрального тела, если они от него находятся на расстоянии в разы превышающем гравитационный радиус.

      Вообще меня забавляет страшилка (вот про это следовало бы написать), что чёрная дыра опасна, что она всё затягивает. На самом деле она никак не отличается от ньютоновского тяготеющего центра такой же массы. Для примера сравним Солнце, как оно есть, и чёрную дыру солнечной массы (размером, ЕМНИП, 3 км). В такую ЧД ещё надо умудриться упасть, т.е. надо очень и очень старательно прицеливаться. Чуть мимо — и ваш избыточный момент импульса не даст вам упасть в такую ЧД. Получается, что пролетев в 5 км (ну ок не в 5 км, а в 100 км) от солнечной ЧД, у вас не будет никаких проблем. Если же вы нацелитесь в 100 км от центра Солнца, то вам капец. Пропадёте даже пролетая не в 100 км, и не в 10000 км, а при 1000000 км от него.

      Вывод: чёрные дыры — самые безобидные объекты во вселенной. Ну по крайней мере ЧД звездных масштабов.


      1. Victor_koly
        03.03.2019 23:52
        +1

        Только Вы можете её не заметить в плотной туманности.


        1. Bronx
          04.03.2019 04:56

          В плотной туманности вокруг неё наверняка будет хорошо заметный аккреционный диск.


        1. TheShock
          04.03.2019 05:06

          А разве её нельзя вычислить по притяжению? То есть определяли ведь сотни лет назад, как планеты должны двигаться вокруг Солнца, так даже крайние планеты системы нашли, то же самое можно вычислить и с черной дырой?


          1. Victor_koly
            04.03.2019 10:17

            По притяжению мы не отличим ЧД от звезды массой 3-5 масс Солнца. Но я согласен с Tyusha в том, что не так уж опасно пролететь от такой ЧД на расстоянии 3-5 радиусов Солнца. Максимум — повышенный рентгеновский фон и гамма-кванты энергией 400-500 кэВ. Правда это я взял «мягкое» гамма-излучение. Если вокруг ЧД будет атмосфера из кислорода и более тяжелых элементов с плотностью атомов кислорода (и б.т.э.) хотя бы как в воде при н.у. — этот диапазон будет поглощаться сильнее, а скажем 800 кэВ — уже слабее.


            1. TheShock
              04.03.2019 10:57

              Но ведь звезду мы будем видеть? В точке Х у нас есть что-то с массой 3-5 масс Солнца. Если это звезда — оно светится как звезда. Если черная дыра — заламывает свет. Так можно определить, что это. Я так понимаю, что мы не знаем еще туманностей в космосе, которые могут звезду закрыть, а все туманности, которые мы знаем — это один атом (к примеру) водорода на километры вокруг.


              1. Victor_koly
                04.03.2019 14:03

                Так заламывает свет что звезда, что ЧД одинаково. Разница будет только в траектории тех фотонов, которые натолкнулись бы на плазму короны белого/голубого субгиганта/гиганта (лень искать инфу о сп. классе звезды взятой мною массы).
                Может учет какой-то квантовой гравитации скажет, что на расстоянии радиуса звезды упомянутой массы грав. поле ЧД более сильно искривляет метрику, но с проверкой любого вида КТГ пока кажется проблемы.


              1. vedenin1980
                04.03.2019 14:16

                оно светится как звезда

                Вроде, нейтронная звезда практически не светится в видимом диапозоне, а некоторые виды таких звезд вообще практически ничего не излучают. Определить, где ЧД, а где нейтронная звезда можно лишь в теории по массе.


                1. Victor_koly
                  04.03.2019 14:54

                  Обычные пульсары излучают радиоволны, а «во время еды» — рентген (как правило — тесные двойные системы). Но, механизм порождения рентгена, ИМХО, включает в себя ондуляторный. А ондулятор может выдавать практически любую длину волны, зависящую от периода магнитного поля, энергии частицы (gamma ~ 1 или gamma ~ (постоянная решетки железа)/(комптоновская длина электрона)) и взаимной ориентации импульса чатсицы с магнитным полем.
                  Это Вам и синхротронное излучение типа СВЧ, и «мягкое» рентгеновское излучение (типичное излучение ондулятора в слабых магнитных полях, при нулевом угле ориентации и gamma ~ 1 будет половиной периода, например — ферримагнитного домена). Может выйти весь диапазон между ними. Другое дело, что одно видит рентгеновский телескоп (скажем на большой высоте, где кислорода концентрация падает и длина «своб. пробега» рентгеновских фотонов растет хотя бы на 70%), а другое — радиоволновой в диапазоне СВЧ (скажем интерферометр с базой в 1000 раз больше радиуса зеркала самого крупного рефрактора).


                  1. trapwalker
                    04.03.2019 15:41
                    +4

                    Если бы мне сказали, что ваш комментарий написан нейронной сетью,. я бы не удивился. Хотя в том, что я ничего не понял, исключительно заслуга моего невежества. Впрочем, было очень интересно.


      1. vanxant
        04.03.2019 00:26
        +1

        Ну не такие уж и безобидные, они умеют внезапно делать пиу-пиу и все вокруг поджаривать, когда им удается зохавать что-нибудь массивное.


  1. Fasterpast
    03.03.2019 19:45

    Спасибо, было интересно почитать. Но «Интерстеллар» — научная фантастика (science fiction), искать там «научность» — дело неблагодарное, ибо см. описание жанра научной фантастики. ЗВ, опять же, научной фантастикой никогда и не являлись.


    1. andrey_ssh
      04.03.2019 14:13

      Что-то с этим утверждением не так.


      1. Fasterpast
        04.03.2019 14:48

        Например?

        Научная фантастика (НФ) — жанр в литературе, кино и других видах искусства, одна из разновидностей фантастики. Научная фантастика основывается на фантастических допущениях (вымысле, спекуляции) в области науки, включая как точные, так и естественные, и гуманитарные науки. Научная фантастика описывает вымышленные технологии и научные открытия, контакты с нечеловеческим разумом, возможное будущее или альтернативный ход истории, а также влияние этих допущений на человеческое общество и личность. (с) Википедия

        Т.е. никакой корреляции с реальной наукой в НФ может и не быть. Придумали, что E = mc^3, выстроили вокруг этого придуманный мир с объяснениями и использованием этого допущения — это НФ, а просто нарисовали, что драконы летают в космосе — фэнтези.


  1. ozeon
    03.03.2019 20:11
    -5

    «Более того, если Вселенная бесконечна, то она бесконечна с самого начала. Бесконечна и почти однородна. И эта картинка (и еще красивый рендер со взрывом, который любят показывать) никакого отношения к реальности не имеет. Нет никакой точки, где был взрыв, который распространялся в окружающую пустоту. Повторюсь, вселенная была почти однородной (и чем дальше в прошлое, тем более однородной).»
    — точняк!!! а «дальше в прошлое»- относительно нашего видения далеких космособытий, она и была и остается однородной… бесконечно однородной, я как то мысль свою пытался выразить в плане понимания «начала» вселенной — с составляющей субчастицы атома она началась, начинается, и там же парадоксально она заканчивается, и это явление повсеместно и бесконечно однородно, как и фундаментальные законы квантовой механики.
    теория же Б.В хоть и безобразна и глупа, но такова она есть, приверженцы ее не глупы, они тоже мыслители.


    1. Tzimie Автор
      03.03.2019 20:12
      +2

      С этим вам на Рен ТВ )


      1. ozeon
        04.03.2019 18:33

        спасибо)) может быть вы и правы не в свою тему влез))


  1. juhomuslim
    03.03.2019 20:42
    -11

    Релятивисты уже настолько погрязли в своей лжи, что скоро по бредовости затмят математиков.


    1. Tzimie Автор
      03.03.2019 20:44
      +5

      Я принципиально аппрувлю все комментарии)


      1. Tyusha
        03.03.2019 20:49
        +5

        Конечно. Этот человек ведь специально только что зарегистрировался на Хабре ради этого комментария — не стерпел лжи релятивистов.

        UPD: Хотя может быть, Владимир зарегистрировался уже не в первый раз, кто знает.


        1. gasizdat
          04.03.2019 08:28

          Ну, хотя бы не долго мучался.


  1. Sdima1357
    03.03.2019 20:52

    Не очень понял с горизонтом событий. Тело любой массы пересекая горизонт событий имеет бесконечную кинетическую энергию? Или нет? Если да, то это немного противоречит закону сохранения энергии системы ЧД, тело. Если нет, то оно может выскочить и обратно. Парадокс однако.


    1. Tzimie Автор
      03.03.2019 20:59
      +2

      1. Не бесконечную
      2. С понятием и сохранением энергии в ОТО все сложно… Об этом будет 2 часть
      3. Пространство время внутри дыры устроено так, что выскочить невозможно вне зависимости от энергии. Под горизонтом сингулярность, к которой вы летите, оказывается не впереди, а в БУДУЩЕМ. То есть как вам надо двигаться, чтобы избежать ПОЛНОЧИ сегодня?


      1. Tyusha
        03.03.2019 21:10
        +1

        Про полночь понравилось. Никогда не слышала такой лаконичной и красивой аналогии.


      1. namazi74
        03.03.2019 21:14

        проблемы ОТО разве не пытается решить Излучение Хокинга?


        1. Tyusha
          03.03.2019 21:18
          +2

          У ОТО нет проблем. :) Кроме проблем с сингулярностью. Но излучение Хокинга не про сингулярность. Да и само излучение Хокинга, если уж говорить в терминах проблем, скорее их добавляет, нежели решает. :) Информационный парадокс и всё вот это.


      1. Sdima1357
        03.03.2019 21:16

        «пространство внутри устроено так»
        С чего бы? Если черная дыра достаточно велика, то как многие утверждают, что наблюдатель даже не заметит разницы. То есть локально оно имеет те же свойства.


        1. Tyusha
          03.03.2019 21:22
          +1

          Локально всё пространство везде: в чёрной дыре, в белой, в «эпицентре большого взрыва» устроено одинаково, о чём нам и говорит принцип эквивалентности. Чёрная дыра порождает нетривиальные глобальные топологии, хотя в любом месте чёрной дыры (окромя сингулярности) с пространством-временем локально «всё в порядке». Наблюдатель падает в чёрную дыру, пролетел горизонт, летел себе дальше, как говорится: ничего не предвещало,… но сингулярность.


        1. Tzimie Автор
          03.03.2019 21:25

          И это никак не противоречит тому, что я сказал!
          Локально все выглядит нормально. А в будущем у вас сингулярность

          Вот так это выглядит в координатной системе Крускала, она наиболее близка к тому, как воспринимает мир наблюдатель, падающий в черную дыру
          en.wikipedia.org/wiki/Kruskal%E2%80%93Szekeres_coordinates


          1. Sdima1357
            03.03.2019 21:52

            Две вселенных и две внутренние области одной дыры в этой диаграмме безусловно многое обьясняет. Правда белые дыры не обнаружены… Видимо прячутся в других вселенных.
            Диаграмма конечно даёт вменяемую математическую модель траекторий, но что там есть на самом деле вроде никто не знает


            1. Tzimie Автор
              03.03.2019 22:03
              +1

              две вселенных возникают в идиализированной картине вечно существующей черной дыры, у которой должна быть белая пара.

              Реальные черные дыры, сформированные коллапсом, выглядят иначе и белой черной дыры и негативного пространства там нет: www.semanticscholar.org/paper/26.2-Schwarzschild%E2%80%99s-Spacetime-2/bbe516a300e3cf212ae6a08783509fb1942b2400/figure/5

              Для вращающихся черных дыр все хуже, kerr metrics является «идеализированной» метрикой вечно существующей черной дыры, а реалистичного решения нет


              1. Sdima1357
                03.03.2019 22:28

                Извиняюсь за надоедливость, но как оценить кинетическую энергию падающего пробного тела на горизонте? Очевидно она прямо пропорциональна массе пробного тела. А какой второй множитель? Если она конечна, то ее можно посчитать???


                1. Tzimie Автор
                  03.03.2019 22:30

                  Это будет во второй части статьи)


                  1. Tyusha
                    03.03.2019 22:47

                    Спойлер (я догадалась, догадалась, только тс-с-с-с...)
                    Энергия Вселенной не сохраняется.


                    1. Tzimie Автор
                      03.03.2019 22:52
                      +1

                      Тише!!! )))


                    1. Victor_koly
                      04.03.2019 00:02

                      Спойлер про Вселенную.
                      Она 5 миллиардов лет расширяется с ускорением, если верить модели LambdaCDM. Странно было бы пробовать описать такую Вселенную сохранением энергии. Получаем явную неоднородность времени.


                      1. Tyusha
                        04.03.2019 00:47

                        на законы сохранения барионного и лептонного заряда действительно нет порождающей их симметрии?

                        SU(2)xSU(3)


                        1. Victor_koly
                          04.03.2019 10:24

                          Ну значит я плохой источник раньше нашел. Но люди все равно пытаются ограничить сверху вероятность невозможных процессов, вроде распада
                          Z -> p + e, mu (нарушение з.с. сразу лептонного и барионного заряда).
                          Да, ещё большее ограничение (то есть более слабое) на распады
                          Z -> mu + anti-tau
                          Z -> anti-mu + tau


                        1. thedeemon
                          04.03.2019 15:24

                          SU(2)xSU(3) это про слабое и сильное взаимодействия как раз. Где тут сохранение лептонного заряда?


                          1. vlad9486
                            04.03.2019 17:13
                            +1

                            В стандартной модели есть локальные калибровочные и глобальные калибровочные симметрии. Из локальных получаются взаимодействия и сохранения, конкретно благодаря SU(2) сохраняется слабый изоспин, а благодаря SU(3) цвет. Из глобальных взаимодействия не получаются, но благодаря им сохраняется число лептонов. Это обычно U(1) (кажется только они), глобальные фазы.


                            1. thedeemon
                              04.03.2019 20:51

                              Ээ… А есть чем подтвердить слова?

                              Насколько мне подсказывает склероз, калибровочные симметрии все локальные. Глобальная симметрия U(1) сидит в основаниях КМ, например, физическому состоянию соответствует не просто вектор Гильбертова пространства, а луч — умножение вектора на комплексное число единичной «длины» дает все то же состояние, т.е. в рамках U(1) сколько хошь умножай глобально, ничего не изменится. Вот если локально U(1) симметрию требовать, там уже есть эффект — получается электрический заряд и его сохранение.


                              1. Tyusha
                                04.03.2019 21:55
                                +1

                                Вы правы, я затупила. Локальная калибровочная инвариантность даёт только сохранение электрического заряда, но не лептонного или барионного.


                              1. vlad9486
                                05.03.2019 12:39

                                Ага, там нужно читать

                                В стандартной модели есть локальные калибровочные и глобальные калибровочные симметрии.


                                1. Victor_koly
                                  05.03.2019 13:24

                                  Симметрии есть, например, ЕМНИП, сильное вз-ие никак не зависит от аромата кварка. Ботомоний имеет энергию выше чармония по 2 причинам:
                                  1. Естественная энергия покоя кварков.
                                  2. Наверное, как и в случае с мюоным водородом, энергия состояния растет с ростом приведенной массы частицы «на орбите». Может не прямо линейно, а скажем у пиона почти 95% массы идет на взаимодействие 2 кварков (правда это выражается через глюоны и вирт. пары «кварк-антикварк»).

                                  P.S. Есть план мюонного коллайдера. Может где-то и был протон-пионный, но пион с временем жизни 26 нс будет сложно родить, выделить и разогнать.


                          1. vlad9486
                            04.03.2019 17:13

                            .


                1. Tyusha
                  03.03.2019 22:46

                  Главный вопрос не «как» оценить, а в какой системе координат. Для удалённого наблюдателя падающая частица остановится на горизонте, и кинетическая энергия у неё будет почти нулевая. В то же время наблюдатель на горизонте воскликнет: о какая чёртова прорва кинетической энергии была у этой частицы, только что промчавшейся мимо!


                  1. Sdima1357
                    03.03.2019 22:59

                    На горизонте нет наблюдателей :(. Но если пробное тело достаточно крупное то будет видно по изменению массы черной дыры


                  1. Sdima1357
                    03.03.2019 23:07

                    Да а с точки зрения удаленного наблюдателя, если оно остановится на горизонте, то его кинетическая энергия исчезнет неизвестно куда


                    1. Tyusha
                      03.03.2019 23:20

                      Понятно куда: падение частицы в ЧД — это неупругое столкновение. Часть кинетической энергии в виде отдачи получит ЧД, но это доля крайне мала, в виду несоизмеримости масс. Остальное (а по факту вся) кинетическая энергия частицы, которую она имела вдали от ЧД, добавится к Mc^2 чёрной дыры. Т.е. масса покоя ЧД увеличится больше, чем если просто приплюсовать массу упавшей частицы. Мораль: энергия сохраняется, никакой магии.


                      1. Sdima1357
                        03.03.2019 23:28

                        Вот как раз и был вопрос, а насколько именно больше. В два раза, в три?
                        Ну а если пробная масса достаточно велика, например ещё одна черная дыра, то откуда возьмётся этот константный фактор и куда он потом денется?
                        Можно конечно сказать, что эта энергия уже была в этой системе из двух тел, но тогда суммарная масса двух черных дыр больше суммы их масс…


                        1. Tyusha
                          03.03.2019 23:37

                          Если частица падала из бесконечности с нулевой скоростью (без кинетической энергии), то конечная масса ЧД — прямая сумма масс.

                          В два раза, в три?

                          Во сколько раз кинетическая энергия частицы на бесконечности больше её массы покоя, во столько и будет.


                          1. Tzimie Автор
                            03.03.2019 23:44

                            Если речь о Bondi или ADM mass и первоначально оба тела были в покое, то суммарная масса равна сумме масс МИНУС масса излученная в виде гравитационных волн


                          1. Sdima1357
                            03.03.2019 23:53

                            Возьмём с Земли кирпич. И утащим его в бесконечность. При этом его масса увеличится, ведь мы вытащили его из потенциальной ямы? Примерно как энергия связи в атомном ядре. А если брать более глубокую потенциальную яму, то масса кирпича/пробной частицы выродится на горизонте. Я правильно понял?


                            1. Tyusha
                              04.03.2019 00:02

                              Масса кирпича не изменится, кирпич — он и в чернодырной Африке кирпич. Увеличится суммарная масса системы «Земля-кирпич», т.к. в неё закачали энергию при вытаскивании кирпича на бесконечность. И ещё раз повторю. Кинетическая энергия зависит от системы отсчёта, поэтому энергия частицы на ГС для удалённого наблюдателя нулевая, т.к. она остановилась возле ГС; для местного (на ГС) наблюдателя — энергия частицы какая-то большая.


                              1. Sdima1357
                                04.03.2019 00:11

                                Суммарная масса системы имеет своих двух представителей: Земля и Кирпич. Увеличатся массы обоих и Земли и и Кирпича. Энергия системы где то должна находиться. Закачали в систему энергию — получите добавочную массу. Вытащили энергию — получите дефект массы. Посмотрите на ядерные реакции.


                                1. Tyusha
                                  04.03.2019 00:27

                                  Энергия системы где то должна находиться.

                                  Никогда не задумывалась над вопросом, куда падающий с крыши кирпич складывает свою нарастающую кинетическую энергию. Он же кирпич у него внутри нет места. :)

                                  Энергия системы существует в виде искрящихся синих молний только в кино. Для нас энергия (масса) — это всего напросто число, характеристика, которую внешний наблюдатель приписывает системе при механических взаимодействиях. В этом смысле энергия существует лишь в воображении того, кто выполняет расчёты. Он её придумал для своего удобства, это же модель.

                                  UPD: А ещё не понятно, как кирпич в процессе падения успевает проинтегрировать действие по всем возможным траекториям, отыскать минимум, и именно согласно ему падать. У него же даже мозгов для этого нет.


                                  1. Sdima1357
                                    04.03.2019 00:34

                                    Пока он не стукнулся, с его массой все в порядке, она постоянная, его масса покоя как бы уменьшится на его кинетическую энергию, то есть сумма массы покоя и релятивистской массы останется как его полная энергия, но при ударе он ее потеряет в виде тепла/ излучения. Суммарная энергия при этом сохраняется, а его масса покоя уменьшится


                                    1. Tyusha
                                      04.03.2019 00:42

                                      У вас путаница в терминах. Масса покоя — это масса покоя, они ни о чего не зависит.

                                      но при ударе он ее потеряет в виде тепла

                                      Нет. Горячий кирпич (как замкнутая система) весит больше холодного.


                                      1. Sdima1357
                                        04.03.2019 00:54

                                        Он остынет. И будет весить меньше. В данном случае гравитация обеспечивает отрицательную потенциальную яму. То есть его измеряемая масса будет его масса в бесконечности минус энергия связи. Вам ниже для аналогии атомные ядра.


                                1. Tyusha
                                  04.03.2019 00:36

                                  del


                                1. Bronx
                                  04.03.2019 05:15

                                  Энергия системы где то должна находиться.

                                  В гравитационном поле, например.


                            1. Victor_koly
                              04.03.2019 00:11
                              +1

                              Подобное происходит с массой нейтронов в большинстве стабильных ядер (а нет, во всех стабильных). Как минимум — энергия конфигурации нуклонов с условием «нуклон №N = нейтрон» оказывается ниже, чем у систему «нуклон №N = протон + где-то болтается электрон + нужно компенсировать импульс антинейтрино».
                              Но части протонов в нестабильных ядрах вообще не везет — либо протон слишком высоко от «хорошей пот. ямы» и может породить нейтрон, позитрон и антинейтрино, либо не так высоко, но может столкнуться с 1s-электроном в реакции
                              p + e -> n +nue
                              Например это происходит с вероятностью выше 10% за 1.25 млрд. лет с атомом калия-40.


      1. Norrius
        03.03.2019 22:12

        Мне всегда было интересно: если при пересечении горизонта событий пространство приобретает свойства времени (все пути внутри ведут в сингулярность, так же как все пути снаружи ведут в будущее), значит ли это в некотором смысле, что время становится похоже на пространство? Например, можно ли двигаться назад во времени (что бы это ни означало внутри горизонта)?


        1. Tzimie Автор
          03.03.2019 22:15

          Смотрите на диаграмму Крускала. Конусы действительно ложатся набок, будущее оказывается не вверху, а вбок, к сингулярности, которая оказывается в будущем. Однако это только с точки зрения стороннего наблюдателя — для локального падающего наблюдателя все по прежнему, пространство трехмерно, время одномерно, «No drama».


      1. stanislavkulikov
        04.03.2019 15:47

        То есть как вам надо двигаться, чтобы избежать ПОЛНОЧИ сегодня?
        Так просто быстрее скорости света. Разве нет?


  1. legolegs
    03.03.2019 21:50
    +1

    Для «большого взрыва» я для себя придумал такую аналогию: возмём пресс с большими (бесконечно большими) поршнем и дном. Поршень прижат ко дну, пространства нет. Откручиваем винт и — бабах — у нас по всему пространству под поршнем одновременно появляется эээ пространство. Осторожно, дальше аналогия перестаёт работать.


  1. legolegs
    03.03.2019 21:51

    А вот у меня вопрос: сингулярность в чёрной дыре зачем придумана? Она точно нужна?


    1. pda0
      03.03.2019 22:02

      Она не придумана, она попала туда из математики, точка, в которой невозможно вычислить значение функции. В случае физики — результат бессмысленный. В школьной физике, где тела моделируются материальными точками вы можете получить сингулярность, вычисляя силу гравитационного взаимодействия между двумя точками сближающимися точками, когда расстояние между ними станет равным нулю.


      1. PastorGL
        03.03.2019 22:23

        Не вспомню где (к сожалению), но мне как-то попадалось утверждение, что сигнулярность нельзя пронаблюдать. Следовательно, физическим смыслом она обладать и не обязана, вполне может оставаться этаким математическим артефактом.



        1. pda0
          04.03.2019 00:06

          У чёрной дыры есть масса, материя составляющая эту массу должна находиться в какой-то форме. Она не может быть просто математическим артефактом.


          1. PastorGL
            04.03.2019 02:49

            Так никто и не спорит, что у дыры наблюдаемые физические параметры таки есть. Но у сингулярности, которая скрыта где-то внутри дыры, они теряют смысл.


            1. pda0
              04.03.2019 03:04

              Понятно. Мы на разных языках говорим. Перевожу: Сингулярность это не волшебный объект, Тот-Который-Нельзя-Наблюдать. Это свидетельство неполноты наших физических моделей. Поскольку чёрные дыры принято считать существующими, получается у нас есть (внутри чёрной дыры) некий реальный объект, описать который известными нам законами природы мы не можем. А те, которыми можем (вроде М-теории) находятся на стадии гипотезы и не понятно насколько физичны.


    1. Tyusha
      03.03.2019 22:09

      Сингулярность — результат решения уравнений Эйнштейна. Тут можно было бы поставить точку, но на мой взгляд не всё так очевидно.

      Во-первых, решения Шварцшильда, Керра и других сделаны в предположении, что метрика стационарна (для Шварцшильда статична). Т.е. это решения для ЧД которые «существовали всегда». А в реальных ЧД это заведомо не так, есть фаза коллапса. Решение с коллапсом не найдено, и вряд ли будет когда-либо найдено. И нельзя быть до конца уверенной, что в таком решении обязательно будет сингулярность.

      Ну и во-вторых, что там с единственностью решения уравнений Эйнштейна? Насколько я знаю, глобальная единственность решения задачи Коши для них не доказана.


      1. Tzimie Автор
        03.03.2019 22:23

        Совершенно верно
        Если для невращающейся коллапсирующей звезды есть численные решения, то для вращающейся их нет (изза возникающих временных циклов, которые не дают возможности применить даже численные методы!)

        Наконец, есть еще один прикол. Помимо ОТО есть теория Эйнштейна-Картана, являющаяся расширением ОТО. Экспериментально они совпадают, экспермиентальная проверка разницы далеко за пределами наших возможностей. Так как вторая теория сложнее, то бритва Окамма вроде как ее вырезает. Но теория Картана решает столько теоретических проблем (и избегает сингулярностей!) что вполне возможно что верна как раз она

        en.wikipedia.org/wiki/Einstein%E2%80%93Cartan_theory


        1. Victor_koly
          04.03.2019 00:17

          Точно за пределами:

          Эффекты неметричности в данной теории являются настолько малыми, что ими можно пренебречь даже в нейтронных звёздах.


          Нужно хотя бы пронаблюдать слияние НЗ заранее и словить грав. волны от этого события.


  1. Vsevo10d
    03.03.2019 21:57
    +1

    Кстати, это одна из причин, почему невозможны статические решения Вселенной — она бы просто стала черной дырой


    И только прочитав это, я понял, что ни хрена раньше не понимал в красоте Вселенной.


  1. valemak
    03.03.2019 22:46
    +1

    Вы перепутали планеты в Интерстеллар.

    На фото в статье изображена ледяная планета доктора Манна, на которой не наблюдалось замедления времени.

    А планета-океан Миллер, где 1 час шёл за 7 лет, находилась гораздо ближе к Гаргантюа:


    1. Tzimie Автор
      03.03.2019 22:51

      Похоже вы правы. Сути правда это не меняет) Объяснение «на планету наложено заклятье» куда более правдоподобно


      1. valemak
        03.03.2019 23:09
        +2

        Сам Нолан не скрывал, что «в некоторых местах мы смухлевали» (цитата не точная). С одной стороны в фильме много несоответствий современной физике, гораздо больше, чем Вы упомянули. С другой стороны, история получилась захватывающей и лично я Нолану все вольности простил. Одна из моих любимейших кинолент последних лет.


        1. REPISOT
          04.03.2019 11:22
          -1

          А я просто ржал, когда смотрел. Уровень реалистичности еще ниже чем в «Гравитации»


          1. valemak
            04.03.2019 11:51

            Про реалистичность в «Интерстелларе», в принципе, говорить не приходится. Хотя расчёт внешнего вида чёрной дыры с гравитационным линзированием, по слухам, привёл Кипа Торна к научным открытиям.

            Но фильм всё равно приобщает к знаниям, его заслуга в том, что неискушённый зритель получает яркое представление о таких понятиях как ОТО, сверхмассивная чёрная дыра, кротовая нора, экзопланеты, 4-мерное пространство-время, колония О'Нила и пр. Во-многих местах создатели фильма соврали, и, тем не менее, на данный момент это самое удачное кино, знакомящее рядового зрителя с вышеперечисленными концепциями.


    1. Tyusha
      03.03.2019 22:53
      +1

      Эта картинка ни о чём не говорит. Не факт, что планета близко к ЧД, она просто находится почти на одной линии «с фотографом». Если бы планета была бы близко к ЧД, как кажется, то это значит, что у неё большое наклонение, а значит она пересекает аккреционный диск, где бы ей «очень поплохело».


      1. valemak
        03.03.2019 23:19
        +1

        В принципе да, если отлететь от планеты Манна подальше и поиграть с фокусным расстоянием, то и в этом случае можно получить фото в стиле «крошечная планетка на фоне огромной звезды».

        Но то, что на планете Миллер плещется водичка, а на планете Манна даже облака замёрзли, уже свидетельствует, что первая поближе к чёрной дыре, чем вторая.


    1. Victor_koly
      04.03.2019 00:25

      Про однозначное соответствие между разницей течения времени и силой гравитации я уже писал.
      Ещё упомяну первую теорему о среднем. Только я её почему-то запомнил в форме функции F из раздела «Замечание».
      То есть где-то на пути между 2 точками будет такая, в которой сила гравитации (градиент от грав. пот. со знаком минус) будет равна разнице грав. потенциалов, деленной на расстояние между точками на направлении градиента.


  1. happy-cat
    03.03.2019 23:58

    Все это хорошо и красиво.
    Только наверняка не только у меня в голове вопрос — а что было ДО. До всего вообще и до БВ.
    Хотя откуда вам знать :)
    Ок, тогда спрошу по другому — в каком пространстве произошел БВ? Что это было, тоже вселенная или что?
    Вот этот момент у меня не укладывается в голове :)


    1. Tzimie Автор
      04.03.2019 00:04

      Перечислю известные мне модели:
      1. Eternal inflation — baby universes это островки обычного вакуума в поле сверхплотного. Ответ: пространство такое же как у нас.
      2. Эволюция черных дыр — каждая вселенная возникает из черных дыр предыдущей (В теории Эйнштейна Картана) — примерно тоже
      3. Циклический Big Rip — тоже наше пространство
      4. Туннелирование «из ничего» — ответ: N/A, до Big Bang ничего не было
      5. Симметрия относительно t: U(t)=U(-t), ответ: понятие «до Big Bang» не определено


      1. happy-cat
        04.03.2019 00:20

        п.п. 1,2,3 подразумевают все же что БВ произошел в некоем Уже существуещем пространстве
        п. 4 вообще не укладывается в голове :)
        п. 5 — …


      1. progman_rus
        04.03.2019 18:31

        И ведь может так быть что пространство-время — это лента мёбиуса и наше очень далёкое будущее это БВ?


        1. Tzimie Автор
          04.03.2019 19:00

          en.wikipedia.org/wiki/G%C3%B6del_metric
          Это точное решение уравнений ОТО
          Но увы, нереалистичное


      1. stanislavkulikov
        04.03.2019 18:45

        А как же версия, предложенная Ниайешем Афшорди, о том, что наша Вселенная является трёхмерной браной, получившейся в результате коллапса четырёхмерной звезды в четырёхмерную чёрную дыру?


        1. Tzimie Автор
          04.03.2019 18:58

          Добавляем)


    1. Tyusha
      04.03.2019 00:09
      +2

      Если не брать в расчёт концепции мультивселенных, когда есть материнская вселенная, а мы — лишь грыжа на ней, то пространство и время возникли при БВ. Соответственно вопрос: «что было до», лишён смысла. Это всё равно что спрашивать: что находится к северу от Северного полюса?


      1. happy-cat
        04.03.2019 00:17

        Мы — это наша вселенная? Вы это имели ввиду?
        Я склоняюсь к тому что вселенные находятся друг в друге — как матрешка (ну естественно с поправкой на размеры). Кстати этот момент был в Люди в черном (кажется 2) в самом конце некие существа играют нашими вселенными как мячиками в гольф :)


        1. Tyusha
          04.03.2019 00:55

          Мы вам не скажем: кто мы… (в сторону: Клюшку номер 9, пожалуйста)


          1. WinPooh73
            04.03.2019 09:25

            – Я слыхивал о вещах похлеще, – сказал Форд. – Где-то я прочел об одной планете в седьмом измерении, которую использовали как бильярдный шар в каком-то межгалактическом баре. Так вот, этот шар загнали в лузу, а лузой служила черная дыра. Погибло десять миллиардов разумных существ.
            – С ума сойти! – ужаснулась Мелла.
            – Зато этот удар принес тридцать очков, – заметил Форд.


            "Автостопом по Галактике"


        1. 1Tiger1
          04.03.2019 03:50

          могу ошибаться но мне кажется что вы подходите к этому вопросу эмпирически. проблема в том что наши чувства и мышление (включая логику) образовались на планете и в этой вселенной. а ваш вопрос находится за пределами системы. эмпирические его не взять, он вне привычного опыта. тут только математика спасает. но если кратко у нас нет никаких наблюдений момента До. мы не можем заглянуть в точку пространства-времени которая находиться вне нашего пространства-времени. условно можно сказать что нашего пространства-времени не существовало до взрыва. пока мы можем только проводить аналоги (как с чд) или делать выводы из математики. но все это лишь возможные варианты. доказать их внутри системы нельзя. тупо нет информации. мы можем найти возможные решения как могла образоваться вселенная идентичная нашей, но не можем выяснить точно как образовалась наша вселенная.


      1. kauri_39
        05.03.2019 17:18
        -1

        Что пространство и время нашей Вселенной возникли 14 млрд лет назад не отменяет вопроса — а что было до этого события? Просто нужно перейти к понятиям вечности и бесконечности, в которые наша однажды возникшая Вселенная не вписывается. Так и вопрос «что находится к северу от Северного полюса?» приобретает смысл, если поместить Землю во внешнее пространство, имеющее в своей координатной сетке своё «северное направление».
        Тогда в вечном и бесконечном сверхпространстве вселенных, соразмерных нашей, должно возникнуть очень много, если мы не исповедуем вселенский геоцентризм в духе Горькавого. Эта идея плодотворна и, главное, проверяемая. Только логика и научные знания…


  1. 411
    04.03.2019 00:24

    Так что черную дыру можно сделать из любого материала, не сжимая его — из воды

    И как это будет выглядеть из воды? И каких размеров будет?


    1. Victor_koly
      04.03.2019 00:33

      Огромных размеров. Но это масса меньше 1 миллиарда масса Солнца (сам не считал сейчас, поверил инфе на Вики). Гравитация на поверхности достаточно большая.


      1. 411
        04.03.2019 01:10

        А разве из этого не следует, что наша вселенная должна быть чёрной дырой? Ну или стать ею в какой-то момент.


        1. Victor_koly
          04.03.2019 11:30

          Любые идеи про Вселенную такие, что не понятно, ч то там придумали физики с сингулярностью.
          Вселенная в период времени t <= tPlanck расширялась экспоненциально и выросла до размера явно больше грав. радиуса. Только 5 миллиардов лет назад ТЭ стала расширять Вселенную с ускорением, то есть это член уравнения можно откинуть до времени t = tрекомбинации.
          Хотя Вы задали правильный вопрос. Но Вселенная очень разрежена. 1 масса Солнца в сфере радиусом 3 км — это не ЧД. Правда и белый карлик нельзя сжать в такой радиус естественным процессом. Масштабируем дальше. 1 миллиард масс Солнца в сфере радиусом 2 950 000 000 км — это ЧД. Это нужно загнать в сферу радиусом с большую полуось Урана миллиард масс Солнца, то есть грубо говоря — «кубик» из прижатых друг к другу Солнц с ребром 1000 диаметров Солнца.
          Ещё в 1000 раз больше — уже больше массы нашей Галактики, а до соседней галактики огромное расстояние (около 50 кпк до ближайшей «не карликовой» галактики). Дальней шее увеличение масштабов Вселенной тоже весьма высоко. В радиусе 1 Мпк есть ещё 2 спиральные галактики, самый далекий объект Местной группы — 3.4 млн. световых лет.
          Дальше масштабы опять растут. Соседнее скопление галактик — от 15 до 22 Мпк (как узнал — весьма большая штука, 1300-2000 галактик). Но все равно выходит не более «1/3 галактики» на кубический Мпк (оценка плотности местного сверхскопления), где галактики как правило очень маленькие, редкие достигают 10-20% массы Млечного пути.
          Финальный уровень масштаба — гигантские войды, галактические «нити» и «стены» (порядка и более 1 миллиарда св. лет).


  1. Razoomnick
    04.03.2019 00:26

    Может, вы сможете ответить на пару вопросов, на которые я не смог нагуглить ответы.
    Почему радиус черной дыры не зависит от положения наблюдателя? На горизонте событий вторая космическая скорость равна скорости света, т.е. досветовой скорости недостаточно, чтобы улететь в бесконечность. Но чем ближе наблюдатель — тем меньшую разность гравитационных потенциалов нужно преодолеть, чтобы до него добраться. Получается, что некоторая точка пространства может быть внутри черной дыры с точки зрения далекого наблюдателя, но снаружи с точки зрения наблюдателя поближе.
    И второй вопрос, как следствие первого. Получается, что информацию из черной дыры можно получить, используя цепочку ретрансляторов, по принципу связи с обратной стороной Луны.
    Я догадываюсь, что это не так, но не понимаю, почему.


    1. Victor_koly
      04.03.2019 00:39

      Если бы не было никакого ОТО, то частица материи могла бы летать по эллиптической орбите, которая в перигелии падала под горизонт, а в афелии могла бы отдаляться в 10 или 100 раз дальше от центра ЧД. Но ОТО говорит, что все такие траектории замкнуты внутри самой сингулярности.
      Смысла придумывать для описания окрестностей ЧД комбинацию «СТО + теория грав. Ньютона» кажется нет, т.к. такая теория описывалась бы наверное «вредным понятием» релятивистской массы, которая есть «равной гравитационной и инертной».
      Более точную теорию уже упоминал — объединение уравнения геодезических и метрики, удовлетворяющей уравнению Эйнштейна.


    1. Tzimie Автор
      04.03.2019 12:06

      Вы задаете правильный вопрос
      Почитайте тут
      про разницу «absolute horizon» и 'apparent horizon"
      en.wikipedia.org/wiki/Absolute_horizon
      en.wikipedia.org/wiki/Apparent_horizon


      1. Victor_koly
        04.03.2019 14:06

        Кажется на GT уже была статья. На более глубоком горизонте перестает работать само ОТО и траектории частиц перестают быть однозначными.


  1. Aquahawk
    04.03.2019 00:40

    О. Вопрос. Можно ведь не набрав орбитальной скорости по параболе выскочить за пределы орбиты и упасть назад. Горизон событий у нас ровно на сфере где орбитальная скорость равна световой, так ведь. Получается можно не достигая скорости света взлететь с черной дыры, набрать в пике высоту выше чем горизонт событий и послать оттуда сообщение. И потом упасть вовнутрь. Или я где- наврал?


    1. Victor_koly
      04.03.2019 11:52

      По формальному определению грав. радиус — это когда вторая космическая равна скорости света. Но само понятие о возможности ввести понятие «вторая космическая» в ОТО чуть сложнее (см. мой коммент над Вашим, на 1 минуту раньше). А с «первой космической» в таком простом определении тоже проблемы. Если мы построим динамики СТО в поле гравитации Ньютона, то формально можно будет приравнять грав. силу (GMm/r2) к центробежной с учетом «поперечной массы»:
      GMm/r2 = gamma*m*v2/R,
      да, я тут написал полный бред, но хотя бы как очень грубое приближение можно взять.
      Более точно — вот тут, первый член в формуле для силы.
      Но этот фокус работает только в том случае, если сила гравитации не растет с ростом «инертной массы», а это нарушало бы принцип эквивалентности.


    1. thedeemon
      04.03.2019 15:47
      +2

      В Ньютоновской теории так и получается примерно, но вот уже 100 лет ясно, что она некорректна, а работает ОТО, где определение горизонта событий не имеет отношения к второй космической, и подняться с него нельзя даже на миллиметр. Там чисто геометрически выходит так, световые конусы все «загибаются внутрь» на горизонте и ниже.


  1. gturk
    04.03.2019 00:42

    Таким образом, увеличивая масштаб любого тела, мы всегда дойдем до состояния, когда гравитационный радиус «догонит» настоящий. Так что черную дыру можно сделать из любого материала, не сжимая его — из воды, ваты, газа.

    А что удержит такой объект от гравитационного коллапса?
    Собственно ЧД возникает не потому что материя плотно упаковывается, а материя плотно упаковывается потому что гравитация пересиливает все остальные силы.
    А так интересная концепция — можно себе вообразить невероятно массивное скопление галактик, которые для внешнего наблюдателя выглядит как ЧД, а внутри состоит из самого обычного вещества, звёзд, газа, планет, они все вращаются вокруг друг друга…


    1. thedeemon
      04.03.2019 15:48

      Наблюдаемая вселенная как раз подходящий размер и плотность имеет…


  1. Lemko
    04.03.2019 00:49

    Увеличим радиус Земли в 10 раз (оставив плотность материи такой же). Такая суперземля будет в 1000 раз тяжелее. Гравитационный радиус, соответственно, тоже увеличится в 1000 раз, а объем внутри гравитацонного радиуса увеличится в миллиард! раз. То есть в 1000 раз больше материи нам надо запихать в миллиард раз больший объем, то есть теперь сжимать материю надо в миллион раз меньше.

    Земля КУБ???? Может в расчетах стоит использовать число «ПИ»?


    1. Aquahawk
      04.03.2019 00:53

      Попробуйте на бумажке. При скейле и вычислении относитеньго роста объема и радиуса пи сократится


      1. Lemko
        04.03.2019 00:58

        Сорри торможу. Вы правы


    1. Razoomnick
      04.03.2019 00:54

      Но не только объем куба вырастет в 1000 раз, если радиус увеличить в 10 раз, объем шара тоже.


    1. KvanTTT
      04.03.2019 02:58

      Даже если использовать, то это не играет никакого значения в данном контексте.


  1. dempfi
    04.03.2019 02:27

    Про чёрные дыри и о том, что же происходят при падении на неё мне больше всего нравится объяснение Сусскинда.



  1. DjSapsan
    04.03.2019 03:05
    +1

    Известно, что никаким экспериментом невозможно отличить ускорение от гравитации. Каждое действие при ускорении можно соотнести с таким же действием при воздействии гравитации. Чему тогда соответствует гравитация за горизонтом событий? Бесконечному ускорению?


    1. Bronx
      04.03.2019 06:50

      Так как гравитация конечна всюду, кроме сингулярности, то и ускорение там тоже конечно. Горизонт событий не связан с величиной гравитации/ускорения, он связан с [не]возможностью коммуникации.


    1. Wesha
      04.03.2019 09:23

      Известно, что никаким экспериментом невозможно отличить ускорение от гравитации.

      Но это только для материальных точек. А в реальности пользуемся тем фактом, что поле гравитации — сферическое, а поле ускорения — линейное: ходим из угла в угол кабины, в которой нас заперли, с отвесом — в поле гравитации отвес везде будет указывать на центр тяготеющей массы, а в ускоряющейся кабине — везде строго "вниз".


    1. WinPooh73
      04.03.2019 09:33

      Отличить нельзя только локально. Если наблюдателю разрешить немного двигаться в стороны, то он легко отличить ускоряющийся в космосе лифт от стоящего неподвижно на земле. Во втором случае при сдвиге на метр вправо или влево угол отклонения отвеса будет меняться, т.к. он показывает точно на центр планеты.


      1. Victor_koly
        04.03.2019 11:55

        Тут вопросо в том, использовали ли отвес при изготовлении пола лифта. Вдруг у него радиус кривизны поверхности 6371 км?


        1. unclejocker
          04.03.2019 15:03
          +1

          Тогда при замерах в ускоряющемся лифте будут получаться «интересные» результаты.


        1. forever_live
          05.03.2019 18:46

          Тогда у кабины будут разные размеры пола и потолка, или наклонные стены, или какие-то ещё искажения геометрии


          1. Victor_koly
            05.03.2019 19:07

            Конечно разные. Например — радиус кривизны поверхности пола будет на 3 метра меньше, чем у поверхности потолка. Стены ровные, то есть под углом скажем 1/6371000 радиан (это почти 0.0324 угловых секунд, сможете измерить без постройки интерферометра из стен лифта?).
            Но я вообще-то не имел в виду такое. Мы используем литье в форме размером скажем 2 на 2 метра, чтобы «отлить» 2 листа жести (одинаковых). Ну то есть не жести, а чего-то более прочного, что не прогнется хотя бы под собственным весом.


            1. forever_live
              05.03.2019 21:28

              Так углы отклонения отвеса будут того же значения, правильно? В обсуждении предполагается, что наблюдатель способен их измерить, иначе этот тред не имеет смысла.


              1. Wesha
                05.03.2019 23:15

                В постановке вопроса о равноценности инерции и гравитации предполагается, что у запертого в лифте наблюдателя есть любые измерительные приборы с фантастической точностью измерения, а вот выйти/каким-либо образом выглянуть из лифта он принципиально не может.


                Поэтому описание лифта Энштейна начинается с постулата, что лифт "висит над какой-то гравитирующей массой в однородном поле" — каковое, как идеальный мужчина, не существует.


              1. Victor_koly
                05.03.2019 23:37

                Конечно способен, но с помощью отвеса. В смысле он может понять, что стены слегка расширяются, но это вроде как не очень важно.
                Но вообще-то смысл замечания к принципу эквивалентности (сильному) у меня в том, что невозможно создать схему ускоренного движения НСО, которая была бы эквивалентна даже такому простому полю гравитации, как поле однородной (или хотя бы описанной плавной функцией ro = ro ( r )) сферы.
                P.S. Под плавной функцией плотности в контексте ОТО будем иметь в виду такой тензор энергии-импульса, чтобы возможные разрывы при расчете тензора Римана устранялись.

                Объясняю, о чем речь
                Вот формула тензора.
                Пуская у нас полностью равны друг другу ков. производные в первых 2 слагаемых для любых пар (ro, sigma) и (mu, nu) в точках, где разрыв функции давал бы бесконечность и аналогично — при суммировании последних 2 произведений по lambda все проблемы вычисления символов Кристоффеля устраняются для любых пар (ro, sigma) и (mu, nu).


  1. IsyanovDV
    04.03.2019 04:15

    Спасибо за статью, тема интересная, сам увлекаюсь и ищу материалы по этой теме. Тема огромная и неисчерпаемая, одних теорий возникновения Вселенной десятки.

    Жду продолжения.


  1. 1Tiger1
    04.03.2019 04:34

    «Когда видимая Вселенная была не больше размера атома» — Была ли на момент взрыва вокруг этой сингулярности пустота, или бесконечное однородное пространство-время, было ли вокруг бесконечное количество подобных пузырьков или вся вселенная это статическая запись на неком абстрактном компьютере/хранилище информации — это вопрос скорее филосовский. У нас нет и скорее всего не будет возможности «заглянуть за грань», получить какую-либо информацию подтверждающую одну из теорий. По крайней мере пока не сможем выйти за пределы «нашей вселенной».
    Я это к чему, эта картинка имеет отношение к нашей реальности. Да БВ не отменяет того что вокруг пузырька что-то существовало, что и как и существовало ли — науке это неизвестно. И вряд ли будет известно. Упираемся в скорость света и стрелу времени. Если рассматривать только нашу вселенную то это не «миф», а рассматривать ее как часть чего-то большего — имхо это уже за пределами теории большого взрыва, как и причины возникновения этого пузырька. Вы верно заметили, только t>0. Да может показаться софистикой, но это вопрос что рассматривать в качестве доступного обьекта изучения.

    Чаще встречается другое заблуждение этой теории в «обывательском восприятии», что материя просто разлетается очень быстро в неком обстрактном и постоянном пространстве и абсолютном времени. Буквально как граната рванула и осколки разлетаются. Сложно представить раздувание пространства, если сложнее раздувание пространства-времени.


  1. Grief
    04.03.2019 05:03

    Насколько малой может быть черная дыра, если пространство все-таки неквантуемо? Какие есть ограничения, помимо необходимости огромного количества энергии для того, чтобы целенаправленным выстрелом на более мощном, чем существующий, коллайдере столкнуть частицы в одной точке и перешагнуть порог плотности в момент столкновения для рождения микроскопической черной дыры для лабораторного изучения?


    1. Tyusha
      04.03.2019 09:13

      Независимо от того, какой окажется физика пространства, спектр масс ЧД ограничен снизу минимальным значение порядка планковской массы (1 нанограмм) и дальше масса дискретно растёт как корень из N (некое квантовой число, описывающее состояние ЧД). Такие результаты получаются даже без теории квантовой гравитации, достаточно теории поля и ОТО.


      ЧД не стабильна к переходу в состояния с меньшим N. Но без квантовой гравитации не может быть решён вопрос со стабильность наинизшего энергетического состояния. Другими слова, остаётся ли "огарок" от ЧД после хокинговском излучения, или же она полностью рассасыаается.


      Ваш вопрос про образование ЧД при высокоэнергетиче ких столкновениях отвечу так: никаких фундаментальных запретов на это нет, и я внутренне уверена, что это возможно. Другое дело, что "нарастить мясо" на такой ЧД не получится из-за её быстрого распада до основного состояния (из-за излучения Хокинга). А сечение взаимодействия малых ЧД с веществом нереально исчезающе мало (нейтрино взаимодействуют с матерей на порядок порядка сильнее). Так что быстро накидать в новорождённую ЧД частиц, чтобы она подросла, вряд ли возможно.


      Поэтому того, что в ЦЕРНе родят ЧД, которая разрастётся и поглотит Землю, бояться не стоит. Даже если там и родится микроЧД, то она просто провалится к центру Земли, где и будет летать по эллиптической орбите, т.к. вещество для неё прозрачно. Ну или улетит в неведомую даль космоса, если у неё окажется существенный импульс.


      1. aghast
        04.03.2019 11:41

        насколько я понимаю, в космосе есть излучение с куда большими энергиями, нежели способен дать коллайдер. и если такое и происходит, то явление не такое уж что и редкое.


        1. Victor_koly
          04.03.2019 12:06

          Столкновение 1.22*109 частиц с энергией 10^10 ГэВ в сфере радиусом в планковскую длину — это по Вашему очень вероятное событие? Может Вы не понимаете масштаб величин.
          Планковская масса — 2.176*10-5 грамм, то есть 1/46 кубического миллиметра воды.
          На LHC пробуют найти сверхмалые ЧД, может они возможны только в многомерных теориях гравитации. В доступном диапазоне энергий (до 10 ТэВ) процесса «испарения» таких ЧД не замечено, может просто статистики не хватает. Это на 6 порядков меньше упомянутой мною выше энергии частицы космических лучей, участвующей в гипотетичесокм рождении ЧД.
          Да, есть ещё 1 проблема. Из протонов Вы такую ЧД не соберете, их оттолкнет электростатический потенциал штуки «0.02 мг протонов» с расстояния «1 длина Планка». Можеет оценить необходимую энергию, это Вам не «слабый» барьер в термояде.


          1. Sdima1357
            04.03.2019 13:05

            А нам не нужно столкновение 1.22*10^9 частиц с энергией 10^10 ГэВ, достаточно столкновения одной частицы с энергией 1.22*10^19 ГэВ с любой другой. Не думаю, что это редкое событие. Видимо если что то и образуется оно неустойчиво и не опасно.


            1. vesper-bot
              04.03.2019 13:49
              +1

              1.22*10^19 ГэВ

              Эммм, это на 11 порядков больше энергии частицы "О майн готт", которая по текущим теориям не должна была иметь возможность пролететь далеко по вакууму без потерь энергии. И что-то мне сдается, что процесс, который генерирует такие о-майн-готт частицы, сложно раскочегарить ещё на 11 порядков вверх.


            1. Victor_koly
              04.03.2019 14:23

              Тут Вы немного ошиблись. Любая статья о физике частиц (на коллайдере в смысле) оперирует понятием sqrt(s), по крайней мере — при кинетической энергии в системе ЦМ в разы больше, чем среднее геометрическое энергий покоя частиц.
              Вот типичный пример применения такого обозначения:
              arxiv.org/abs/1609.05122
              Но, протон с кин. энергией 1 ПэВ (а ещё лучше — антипротон) — это уже интересный объект, т.к. его столкновение с любым протоном, кроме летящего «назад» породит в направлении «вперед» огромную струю частиц, в основании которой могут рождаться даже t-кварки. И т.к. импульс у частицы огромный, а в системе ЦМ ипульс нулевой, значит в этой струе суммарный «продольный» импульс частиц должен быть 1 ПэВ/c (включая всякие бозоны, родившиеся в процессе). Именно столкновение таких частиц с легкими атомами (водород, углерод, азот и кислород) в атмосфере фиксируется детекторами площадью десятки гектар.


  1. funca
    04.03.2019 07:42

    Что в центре черной дыры? Может-ли там оказаться комфортная для жизни невесомость?


    1. Tzimie Автор
      04.03.2019 12:10

      Свободно падающий наблюдатель всегда ощущает невесомость. Она комфортна если приливные силы невелики. Для невращающейся черной дыры все плохо, как уже говорилось, как ни рыпайся, все кончишь в сингулярности.

      Для вращающейся есть решения комфортных орбит — но для «идеального» решения Керра. Что внутри «реалистичных» — черных дыр — никто не знает.


      1. Static_electro
        04.03.2019 17:17

        Где-то читал, что пересечение горизонта событий практически эквивалентно полному разрушению любого объекта, так как «двигаться можно только к сингулярности» (неточная цитата) и взаимодействие между любыми частицами этого объекта, направленное «наружу», пропадет. Т.е. например, никаких нервных импульсов.
        Что с этим мифом?


        1. Tzimie Автор
          04.03.2019 17:18
          +2

          Как я уже писал, свободно падающий наблюдатель может даже не заметить пересечения горизонта
          No drama!


        1. vedenin1980
          04.03.2019 17:24

          В статье же вроде есть опровержение мифа:

          стало ясно то, что позже назвали «No drama» — при пересечении горизонта событий ничего особенного для свободно падающего наблюдателя не происходит — никакой тревожной музыки, пиу-пиу и зеленоватых всполохов. Провал под горизонт можно даже не заметить!


          С точки зрения падающего наблюдателя горизонта событий не существует, он просто притягивается к массивному телу, локально все точки пространства до сингулярности доступны, поэтому ходить по звездолету он сможет без проблем.


          1. Static_electro
            04.03.2019 17:32

            Я, вероятно, совершаю ошибку, пытаясь осознать то, что нужно просто принять :)
            Но неужели визуально это будет незаметно? Правильно ли представлять горзионт «снаружи» как черный непрозрачный шар? И если да — то как он будет выглядеть по мере приближения к нему (и — пересечении)?


            1. vedenin1980
              04.03.2019 17:44

              Но неужели визуально это будет незаметно? Правильно ли представлять горзионт «снаружи» как черный непрозрачный шар? И если да — то как он будет выглядеть по мере приближения к нему (и — пересечении)?

              Правильно. В теории, как я понимаю, для наблюдателя он будет падать к черному шару, при этом визуально шар будет уменьшаться (он будет видеть ближащие внутрености ЧД перед собой, так свет вполне сможет пройти это расстояние). Момент перехода горизонта событий никак не будет заметен, как мы не замечаем того что большая часть наблюдаемой Вселенной уже для нас недостижима на скорости света. Наблюдатель будет видеть то что находится внутри ЧД по мере полета к сингулятрности (например, другой звездолет безуспещно пытающийся вылететь из ЧД). То что с ним будет при достижении сингулятрности — никто не знает.

              Есть теория, что вся наша наблюдаемая Вселенная это и есть внутрености ЧД (в которой пространство расширяется), в этом случае наблюдатель проживет без проблем всю свою жизнь внутри ЧД.



        1. Victor_koly
          05.03.2019 10:03
          -1

          В принципе Вы правы в таком смысле. Скажем обычное твердое тело существует за счет электромагнитных сил. ЭМ сила вызывается виртуальными фотонами. Каким бы виртуальным не был фотон, мы не сможем посчитать пропагатор в направлении, куда вообще не могут вести геодезические (я честно не учил КТП, так что предполагаю только). По этому, условно говоря, каждое упавшее под сингулярность ядро атома перестанет притягивать окружающие электроны, оставшиеся над горизонтом. Или скажем упадут в железе таким образом только ядро и внутренние электронные оболочки (конфигурация аргона). Электроны 3d6 4s2 останутся болтаьться в электронном газе.
          Но это конечно прикол, так как до падения в сингулярность столкновения в плазме должны ионизировать оставшийся атом «аргона» полностью. Весьма сильная ионизация начнется уже при kT = 68.6 эВ, электроны 1s потребуют может в 9 раз большей энергии.


  1. devlev
    04.03.2019 08:46

    К вопросу о расширяемости:

    Нашей вселенной это не грозит, так как она расширяется.
    А как вы относитесь к этой теории habr.com/en/post/371363


    1. Tzimie Автор
      04.03.2019 12:13

      Начало про уменьшение массы верное. Дальше — мне кажется малоправдоподобно, но конкретная критика пусть исходит от профессиональных ученых.


  1. tretyakovpe
    04.03.2019 10:22

    Нет ли теории, что наша вселенная это «обратная сторона» сингулярности чёрной дыры?
    Ну т.е. если провалиться в сингулярность, то попадёшь в новую вселенную, для которой эта сингулярность — большой взрыв.


    Увидел комментарий ТСа


    1. Tzimie Автор
      04.03.2019 12:14

      да, насколько я помнб это теория Leonard Susskind


  1. irnis
    04.03.2019 11:43

    > Никто не судит Звездные войны за звук взрывов в космосе.

    более того — есть вполне научное объяснение: взрывы порождают поток энергетических частиц и/или волн, которые взаимодействуют с корпусом корабля и уже это создает звуки внутри корабля.


    1. Victor_koly
      04.03.2019 12:09

      Ионные пушки весьма рулят в реальности «Зв. войн», насколько я помню.


    1. forever_live
      05.03.2019 19:02

      Даже в этом случае должна быть задержка между вспышкой и звуком


  1. S_h_e_l_d_o_n
    04.03.2019 12:15

    Возможно уже был такой комментарий. Сорри если повторюсь.
    Главный герой не пересекал горизонт событий. Он попал в эргосферу. Это керровская чёрная дыра.
    Источник — девушка, которая писала формулы к фильму.


  1. Artemis86
    04.03.2019 12:15
    -1

    Очень лекго и просто сочинять теории, которые невозможно проверить. Многие сделали на этом себе имя. Пора бы человечеству поумнеть и научиться отличать фантазии от реальной науки.


  1. berrics
    04.03.2019 12:15
    +1

    полуправды и неверных выводов. Начинается все с правильного утверждения: при приближении к горизонту событий для наблюдателей, подвешенных над черной дырой, замедление времени стремится к бесконечности. Это верно.

    с правильного утверждения:

    Вы серьёзно? Обсуждая вообще гипотетические штуки вроде черных дыр (неизвестно, существуют ли они, по крайней мере, с теми свойствами, какие к ним приписываются), называть что-то верным или неверным. Ну, даёте народ…


    1. Artemis86
      04.03.2019 12:57
      -2

      Вот-вот. О чём я и сказал выше. Люди создали себе некую псевдонаучную матрицу и живут в ней, при этом ещё и пытаются спорить и защищать свой виртуальный мирок от нападок «конкурентов». Мне это сюжет их психбольницы напоминает. Куда-то не туда человечество движется. Ну по крайней мере довольно большая его часть.


      1. Artemis86
        04.03.2019 14:49

        Фанатам Эйнштейна и всевдонаучным троллям не понравилось моё мнение:)


        1. mrhru
          04.03.2019 20:02
          +3

          "Ничего не понимаю, значит это неверно".
          Фанатам Эйнштейна и всевдонаучным троллям обычно не нравятся воинствующие неучи.


          1. Artemis86
            05.03.2019 02:12
            -4

            Ооо, лепить ярлыки вы мастера)
            Вот вам ссылочка:
            www.ikz.ru/~smulski/Papers/EssenceSR2r.pdf


            1. mrhru
              05.03.2019 05:40
              +1

              Ну, да, Вы не одиноки.


        1. TheShock
          04.03.2019 20:10
          +3

          не понравилось моё мнение
          Не путайте мнение и невежество. Если я скажу, что Луна и Солнце — это огромные прожекторы, а звезды — это пробитые в покрывале дырочки, то это будет не мнение, а невежество.


          1. Artemis86
            05.03.2019 01:41
            -5

            А если я скажу что Эйшнтейн в школе был заурядным троечником, потом работал клерком патентного бюро, был мужем талантливой сербки Милевы Марич, 20 раз безуспешно выдвигался на получение Нобелевской премии и получил её с 21-й попытки, и вовсе не за СТО, как некоторые невежды думают, а за исследование фотоэффекта, открытого русским учёным Столетовым? Это вы как назовёте?


            1. TheShock
              05.03.2019 01:45
              +4

              Это вы как назовёте?
              Я назову это «высокомерием». Сравниваете себя с Эйнштейном? Серьёзно?


            1. saboteur_kiev
              05.03.2019 02:59
              +4

              Я скажу, что вы не умеете читать вообще, и биографию Эйнштейна в частности. Кем-кем, а заурядным он в школе не был, и ВСЕГДА был одним из выдающихся учеников.

              А то, что некоторые двоечники не удосужились прочитать про то, что в определенный период обучения в учебном заведении была «обратная» шкала, где единица считалась высшим балом, то что Эйнштейн ненавидел зубрежку из-за чего у него были конфликты с учителями — это проблема тех двоечников, которые не умеют читать.


              1. Artemis86
                05.03.2019 04:37
                -3

                Вы так самоуверенно это утверждаете, словно учились вместе с ним в одном классе. Вам не кажется что такая самоуверенность это признак некого душевного дисбаланса?


                1. saboteur_kiev
                  05.03.2019 16:03
                  +1

                  Эйнштейн родился не в средние века.

                  Его реальные аттестаты вполне доступны для общества, его биография подтверждена и проверена многочисленными журналистами и историками.

                  А вот вы почему так самоуверенно считаете, что он был двоечником?


            1. Bronx
              05.03.2019 05:46
              +1

              А если я скажу что Эйшнтейн в школе был заурядным троечником, потом работал клерком патентного бюро, был мужем талантливой сербки Милевы Марич, 20 раз безуспешно выдвигался на получение Нобелевской премии и получил её с 21-й попытки

              Вы так самоуверенно это утверждаете ©, словно учились вместе с ним в одном классе, работали в одном патентном бюро, вместе были мужем Милевы Марич, вместе выдвигались на Нобеля и вместе её получали. Вам не кажется что такая самоуверенность это признак некого душевного дисбаланса?


              1. Artemis86
                05.03.2019 12:11
                -2

                Верующие люди не зря говорят что дьявол-обезьяна Бога.


                1. gdsmiler
                  05.03.2019 12:15
                  +1

                  Вы действительно не находите в Ваших комментариях ничего ироничного?


                1. Victor_koly
                  05.03.2019 13:31
                  +1

                  Для искушения человека сначала Бог создал Дьявола, потом Коперника, а ещё позже — Королева с Гагариным. И ещё фон Брауна:

                  Эффективность боевого применения «Фау-2» была крайне невысокой: ракеты имели малую точность попадания (в круг диаметром 10 км попадало только 50 % запущенных ракет) и низкую надёжность (из 4300 запущенных ракет более 2000 взорвались на земле или в воздухе при запуске, либо вышли из строя в полёте[источник не указан]). По различным источникам, пуск 2000 ракет, направленных за семь месяцев для разрушения Лондона, привел к гибели свыше 2700 человек (от каждой ракеты погибал один или два человека).


                  Так что в сравнении с изобретателем газовой камеры фон Браун почти святой.


        1. vlad9486
          05.03.2019 12:52
          +1

          Здесь обсуждается не Эйнштейн, а ОТО. Личность Эйнштейна с истинностью ОТО вообще никак не связанна.


    1. thedeemon
      04.03.2019 15:54
      +3

      Достаточно добавить «в рамках данной теории». Есть математически проработанная теория, хорошо проверенная там, до куда мы смогли дотянуться, и есть вполне конкретные ее предсказания. Вот о них и речь.


  1. VasikAlexey
    04.03.2019 12:39

    Очень сильно верю, что описанные в третьей книги «Сказаний о прошлом Земли» методы ведения войны путём уменьшения скорости света и понижения размерности солнечной системы случатся не на нашем веку.
    Хотя экранизацию гляну :)


  1. AlexAV1000
    04.03.2019 12:52
    -3

    Весна. У программистов баз данных началось обострение. Оказывается, в свободное время, программисты баз данных, ну чисто как хобби, изучают ОТО и физику элементарных частиц. А меня всё мучал вопрос, кто же даёт инвайты на Хабр разным фрикам?


    1. solariserj
      04.03.2019 13:28
      +1

      > кто же даёт инвайты на Хабр разным фрикам

      В споре рождается истина. (с)

      А так в начале 20 века тоже считали что в Физике всё открыто, и если бы не фрики(как вы их называете) придумывали различные теории,(от простых до абсурдных) то долго бы топтались на месте.


    1. Tyusha
      04.03.2019 14:18

      Как я понимаю, Tzimie весьма глубоко изучил ОТО и физику элементарных частиц задолго до баз данных. SQL — всего лишь ремесло, дающее хлеб насущный. Квалификация автора в физике видна невооружённым глазом.


      1. Victor_koly
        04.03.2019 14:28

        Я конечно ОТО глубоко не изучал, вывод выражения для действия грав. поля не проведу сам.
        Физику эл. частиц изучали чуть лучше, но все равно не особо разбираюсь пока. Скажем в нейтроне действительно есть W-бозон массой в разницу масс нетйрона и протона.


    1. akura13
      04.03.2019 20:48
      +2

      Все верно. Я такой. И интересы даже совпали. Не ругайте нас. Нам очень нравится ОТО и физика элементарных частиц.


  1. dolovar
    04.03.2019 12:55

    если Вселенная бесконечна, то она бесконечна с самого начала. Бесконечна и почти однородна.
    Эти два постулата являются предположениями и потому не могут опровергнуть что бы то ни было. Тем более, что в конце статьи пришли к «невозможны статические решения Вселенной».
    Приливные силы разрывают всякого еще на подлете к черной дыре… есть сверхмассивные черные дыры, массой в миллиарды солнечных масс, а радиусом больше орбиты Урана.
    Это не опровержение мифа, а уточнение — есть предположения о возможности существования некоторых объектов, при падении в которые, теоретически, некоторые тела не будут разорваны.

    IMHO, главный миф физики скрывается за словом «доказано». Если ознакомиться со статьями Элиезера Юдковского и других рационалистов, если освежить знания о научном методе, то реже возникает желание махать шашкой с постулированием невозможности чего бы то ни было. Любой известный закон является лишь ступенькой на пути к пониманию реальности — мы придерживаемся тех теорий, которые помогают продвинуться дальше. Любые наши знания являются временными.

    А в данной статье даже не теории, а гипотезы выдаются за часть реальности, чтобы опровергнуть… сценарий кинофильма?

    Я решил сформировать список наиболее часто встречающихся неправд и полуправд. Итак, самое частое это…
    Самое частое в какой из выборок?


    1. Tzimie Автор
      04.03.2019 13:12

      >миф физики скрывается за словом «доказано»
      В физике ничего никогда не доказано, так как любой эксперимент может обрушить любую теорию, однако любое число подтверждающих экспериментов не делают теорию «доказанной». Однако, когда теория достаточно устоялась то начинает действовать правило «extraordinary claim requires extraordinary evidence».

      Буквальное отношение к слову «доказано» — еще одна проблема папулярной науки

      >Самое частое в какой из выборок?
      Больше всего по заблуждениям новичков приходящих на physicsforums.


      1. dolovar
        04.03.2019 13:47

        Буквальное отношение к слову «доказано» — еще одна проблема
        Если вы понимаете проблему, то почему в вашей статье встречаются безапелляционные заявления, в которых одно из предположений выдается за часть реальности, позволяя откинуть другие предположения в список мифов? Например:
        Нет никакой точки, где был взрыв, который распространялся в окружающую пустоту. Повторюсь, вселенная была почти однородной
        Если ТБВ говорит лишь о том, что было после БВ, то почему отсутствие единой точки или точек до БВ, а также предположения о состоянии вселенной до БВ превратились в доказанный факт?

        Замечу, что я не придерживаюсь обратной точки зрения, не пытаюсь доказать преимущество предположения «была одна точка» — оно не моё, говорю не о нём. Но для отнесения чего-либо к мифам нужно нечто большее, чем гипотеза «возможно было иначе». Для развенчания «наиболее часто встречающегося заблуждения» хотелось бы увидеть более основательное раскрытие темы.


        1. Bronx
          04.03.2019 14:39

          почему в вашей статье встречаются безапелляционные заявления, в которых одно из предположений выдается за часть реальности

          Чего люди часто не понимают, это то, что физика не "выдаёт нечто за реальность", а "объясняет реальность через модели". Если нечто "безапеляционно утверждается", то это всегда утверждается в рамках некоторой модели.


          почему отсутствие единой точки или точек до БВ, а также предположения о состоянии вселенной до БВ превратились в доказанный факт?

          ТБВ — это физическая модель (одна из многих), и конкретно в этой модели дела обстоят именно так, что позволяет модели объяснить определённые наблюдаемые факты. Вы можете предложить другую модель, где будет единая точка начала или несколько точек, и будут другие предположения. Если это будет объяснять наблюдения — это будет прекрасно. Если это будет объяснять больше наблюдений, чем все остальные модели — просто расчудесно.


          1. Victor_koly
            04.03.2019 14:57

            Мне срочно нужно дополнить модель космологии уравнением состояния для ТМ (вместо банального P (ro) = 0) и проверить её на масштабе от конца инфляции до эпохи рекомбинации.


          1. dolovar
            04.03.2019 14:59

            физика не «выдаёт нечто за реальность», а «объясняет реальность через модели»
            Не спорю и вполне поддерживаю, но не сильно ли мы отклонились от обсуждаемой статьи? Я про статью говорил, в ней есть что-то вроде:
            Наиболее часто встречаемая неправда или полуправда — «когда Вселенная была не больше размера атома». Правда — нет никакой точки, где был взрыв, который распространялся в окружающую пустоту.
            Как это соотносится к «объясняет реальность через модели»? Я вижу утверждение, а не объяснение.

            ТБВ — это физическая модель (одна из многих), и конкретно в этой модели дела обстоят именно так, что позволяет модели объяснить определённые наблюдаемые факты.
            Я не спорю и вполне поддерживаю. Но в обсуждаемой статье сказано примерно так:
            ТБВ это НЕ теория о большом взрыве, а о том, что случилось после большого взрыва. До БВ не было никакой точки, где был взрыв, и вселенная была бесконечной и однородной.
            И меня смущает то, что для опровержения мифа упоминается только теория, которая, судя по упоминанию же, для этого не подходит. В статье не сказано ничего о том, на чем основаны постулаты.

            Вы можете предложить другую модель
            Нет, конечно, я лишь комментировал конкретную статью. Именно в статье есть то, что выбивается из взвешенного подхода, который озвучен в комментарии ниже:
            Физики как раз довольно скромны и понимают, насколько они далеки от познания мира.


            1. Bronx
              04.03.2019 23:11
              +1

              Ну, возможно, утверждения следует чаще сопровождать пометкой «в рамках данной модели», чтобы у людей не было впечатления о том, что «физики вещают истины», а не обсуждают модели. Однако как часто это делать, чтобы никто 100% не возмутился?


              1. dolovar
                05.03.2019 15:46

                в рамках данной модели
                Еще раз предлагаю в обсуждении вернуться к обсуждаемой статье.
                О рамках какой модели идет речь в тексте после первого же подзаголовка «Большой взрыв»?
                Упомянута только ТБВ, но она не подходит, поскольку говорит о времени послевзрывия, о чем сказано в статье же.

                Данная статья про мифы. Самым частым назван миф о большом взрыве из одной точки.
                Для развенчания мифа выдвинуты постулаты — до БВ Вселенная бесконечна и однородна, не было точки, из которой взрыв распространялся в пустоту.
                Вопрос — о какой модели идет речь?
                Ответ — о какой-то неназванной. Что-то неназванное опровергает «взрыв из одной точки», а также постулирует «до взрыва бесконечна и однородна».
                Мне кажется, что какой бы то ни было миф нельзя опровергнуть постулатами, основанными на чем-то неназванном.


                1. Victor_koly
                  05.03.2019 17:07

                  Я вот скажем скажу, что после инфляции радиус Вселенной стал 10^5584000 метров. Действительно ли меня в такой теории должен интересовать размер Вселенной до инфляции? Или мне придется назвать такую Вселенную Мультивселенной (либо наоборот — назвать «нашу Вселенную» Метагалактикой)?


                  1. dolovar
                    05.03.2019 17:14

                    Извините, но я не вижу связи вашего комментария с обсуждаемой статьей или с моим комментарием, который относился к конкретной статье.

                    Если я скажу, что после инфляции радиус Вселенной стал 10^5584005 метров, то это будет не теория, а высказывание. Высказывание может породить интерес к размеру до инфляции, может не породить. В любом случае, моё или ваше высказывание не позволяет назвать мифом чье-то (не моё) высказывание «Вселенная взорвалась из одной точки ничтожно малого радиуса».


                1. Bronx
                  06.03.2019 04:19

                  Упомянута только ТБВ, но она не подходит, поскольку говорит о времени послевзрывия, о чем сказано в статье же.

                  ТБВ принесла (вольно или невольно) сам термин "Большой Взрыв", поэтому если говорим о "Большом Взрыве" — значит говорим о ТБВ, очевидно. Да, ТБВ не объясняет, почему этот "взрыв" произошёл и объясняет лишь то, что происходило при t > 0. Но если устремить время к 0, то никакого "центра взрыва" в этой модели так и не появляется, поэтому в рамках данной модели "взрыв из точки" — это миф, порождённый неудачным термином.


    1. TheShock
      04.03.2019 13:28

      IMHO, главный миф физики скрывается за словом «доказано».
      У многих теорий есть границы применимости. Физики делают наблюдения, стараются их обобщить, строят гипотезы, стараются прийти к теории, понимают насколько мы слаборазвитая раса, когда смотрят на ТО и КМ и имея эту базу наблюдений строят предположения, что скорее всего будет, если залететь в черную дыру.

      Конечно, никто не знает на самом деле и не узнает пока не залетит. Может вообще сервак, на котором наша вселенная запущенна упадет с NullPointerException.

      Физики как раз довольно скромны и понимают, насколько они далеки от познания мира.


    1. Victor_koly
      04.03.2019 14:33

      есть сверхмассивные черные дыры, массой в миллиарды солнечных масс, а радиусом больше орбиты Урана


      Просто физика не знает сил, которые удержали бы массу в 1 миллиард масс Солнца за пределами сферы радиусом с большую полуось орбиты Урана. Но при этом видит гравитационное влияние таким массивных тел на окружающие звезды.
      Конечно тут нужны уточнения, скажем возле нашей центральной ЧД нашли спустник массой где-то 1300 мас Солнца. Как минимум — это дает ограничение на радиус центральной «а вдруг не ЧД». Дает ли это ограничение сказать «совокупность нейтронных звезд массой 4 миллиона масс Солнца не может быть втиснута в сферу радиусом 11.8 миллионов км без коллапса» — это я не знаю.


  1. SergeiMinaev
    04.03.2019 15:18

    Я ни разу не противник англицизмов, но вот скажите, чем «oversimplification» лучше «переупрощения»?


    1. Tzimie Автор
      04.03.2019 15:21

      тем что слово «переупрощения» я никогда не слышал
      думаю, правильно сказать «слишком сильное упрощение», но это длинно


      1. SergeiMinaev
        04.03.2019 15:29
        +1

        Чрезмерное, излишнее.


  1. floydback
    04.03.2019 15:19

    Я люблю научно-популярные передачи, и все что вы написали говорилось в них 10-20 лет назад. Не очень понятно, что и кого вы тут «опровергаете».


  1. botyaslonim
    04.03.2019 16:18

    Чёрные дыры — это замечательно, спасибо! Но в физике полно всяких приколов и ближе к земле. Например, контрруление на мотоцикле. Будет об этом?


    1. Tzimie Автор
      04.03.2019 17:13
      +3

      напишите


  1. kauri_39
    04.03.2019 16:19
    -5

    «теория Большого взрыва это НЕ теория о большом взрыве (t=0)… Она не отвечает на вопрос, в частности, почему этот большой взрыв произошел.»

    Если можно, расскажите в следующий раз о таких теориях. Например, о теории вечно пульсирующей Вселенной Николая Горькавого. Правда, у его Вселенной есть центр, но нет того, что расширяется — кроме пустоты с абстрактной метрикой пространства-времени.
    В других теориях наверняка есть более ощутимая среда, которая и взорвалась при своём образовании, а теперь при расширении раздвигает скопления галактик. Только среда для этого должна быть плотной…


    1. Victor_koly
      04.03.2019 18:00

      Раздвигает скопления среда?
      Мне вот нравится «абстрактная метрика» как свойство пространства, правда под «вторым горизонтом» ЧД это понятие уже будет плохо работать.


      1. Tzimie Автор
        04.03.2019 18:04
        +2

        И даже непонятно, есть ли этот горизонт. В идеальной Керр метрике есть, но внутри находятся замкнутые временные циклы. А квантовая механика вроде активно сопротивляется возникновению временных циклов — при приближении геометрии к временному циклу начинается генерация ливня частиц, масса которых меняет геометрию и не дает сформироваться горизонту. В общем, тут пока все непонятно


        1. Victor_koly
          05.03.2019 10:15
          -3

          Квантовая механика с неопределенностью — это единственный способ решения временного цикла вида «решил человек убить свою биологическую мать за год до своего рождения». Только Вам может не понравится мир в радиусе 1 световой год от точки «решения парадокса». Ведь главным единственным постоянным условием в «этом проклятом мире» (© Bethesda) будет сам факт возникновения «пути энергии и информации в прошлое» (какой бы физический принцип этого не требовал). А может возникнет несколько таких «проходов» и по сравнению с таким сюжетом «Терминатор: Генезис» покажется очень вероятным развитием событий.


      1. bromzh
        04.03.2019 19:46
        +5

        Раздвигает скопления среда?

        Сейчас этот товарищ начнёт про эфир вещать (это он всё раздвигает). И про то, что вакуум — неподходящее название для вакуума, и из-за этого лингвистического косяка в физике проблемы. То ли дело — его теории. Естественно, без формул, ибо не барское это дело, математикой обмазываться.


  1. tlv
    04.03.2019 16:24
    +6

    Для того, чтобы замедление времени было бы таким, планета должна была быть на расстоянии чуть более 0.003% от радиуса Шварцшильда.


    У Кипа Торна есть целая книга (Интерстеллар: наука за кадром), в которой он рассказывает, как натягивал сову на глобус в разных аспектах.

    Посмотрите её — там хоть и очень натянуто, но здравому смыслу ничего не противоречит. В частности, про замедление времени (дальше прямая паста почти целой главы из книги. Можете глянуть, чтобы увидеть: действительно, очень натянуто, но может.)

    Когда Кристофер Нолан сказал мне, какое замедление времени на планете Миллер ему нужно – один час там на семь земных лет, – я был ошарашен. Я полагал это невозможным, о чем и сказал Крису. «Это не обсуждается», – отрезал он. Что ж, не в первый и не в последний раз я отправился в раздумьях домой, сделал кое-какие расчеты и… нашел выход.
    Я обнаружил, что если планета Миллер будет настолько близко к Гаргантюа, насколько это возможно без риска упасть в черную дыру[36], и если скорость вращения Гаргантюа будет достаточно высокой, замедление «один час за семь лет» возможно. Но Гаргантюа должна вращаться чертовски быстро.
    Для скорости вращения черных дыр есть предел. Если он будет превышен, горизонт событий исчезнет, оставив на виду у всей Вселенной обнаженную сингулярность. А это, по всей видимости, противоречит законам физики (см. главу 26).
    Выяснилось, что для замедления, которое нужно Крису, Гаргантюа должна вращаться со скоростью, близкой к предельной, меньше ее примерно на одну стотриллионную долю[37]. В Кип-версии я по большей части исхожу из этого значения.

    [...]
    В 1975 году я обнаружил механизм, с помощью которого природа предохраняет черные дыры от превышения предельной скорости вращения: когда скорость близка к предельной, черной дыре сложно захватить объект, который летит по орбите в ту же сторону, что вращается она, и который, будь он захвачен, увеличил бы скорость ее вращения. Однако черная дыра с легкостью захватывает объекты, летящие в сторону, противоположную направлению ее вращения, то есть те объекты, захват которых уменьшает скорость вращения черной дыры. Поэтому черная дыра легко замедляется, как только скорость ее вращения приближается к предельной.
    В моем тогдашнем исследовании я уделил особое внимание газовому диску (он напоминает кольца Сатурна), который вращается в одном направлении с черной дырой. Этот диск называется аккреционным (см. главу 9). Силы трения в диске вынуждают газ постепенно, по спирали, переходить в черную дыру, увеличивая скорость ее вращения. Кроме того, трение нагревает газ, и он излучает фотоны. Завихрение пространства вокруг дыры захватывает эти движущиеся по ходу ее вращения фотоны и отбрасывает их прочь, из-за чего они не могут попасть внутрь. И напротив, завихрение захватывает фотоны, которые движутся в сторону, противоположную вращению дыры, и засасывает их внутрь, где они замедляют ее вращение. В итоге, когда скорость вращения черной дыры достигает 0,998 от предельной, устанавливается баланс, при котором замедление за счет захваченных фотонов в точности компенсирует убыстрение за счет поступающего в дыру газа. По-видимому, этот баланс довольно устойчив, и в большинстве случаев можно ожидать, что скорость вращения черной дыры не превышает 0,998 от предельной.
    Однако я могу вообразить ситуации – очень редкие, если вообще встречающиеся в реальной Вселенной, и все же возможные, – когда скорость вращения подходит к предельной гораздо ближе, и даже настолько близко, насколько это требовалось Крису, чтобы замедлить время на планете Миллер: скорость на одну стотриллионную долю меньше предельной. Маловероятно, но возможно.
    В кино это не редкость: чтобы снять шедевр, режиссер часто доводит все до предела. В фэнтезийных фильмах вроде «Гарри Поттера» этот предел находится далеко за границами научной достоверности. В научной же фантастике он, как правило, остается в границах вероятного. Собственно, это главное отличие между фэнтези и научной фантастикой. «Интерстеллар» – научная фантастика, а не фэнтези. Сверхбыстрая скорость вращения Гаргантюа с научной точки зрения возможна.



    1. Tyusha
      05.03.2019 11:11

      Спасибо. Очень интересно. Надо почитать.


  1. Linloil
    04.03.2019 16:35
    +1

    True or false?
    1) Внутри горизонта ЧД есть стабильные орбиты по которым можно вращаться вокруг сингулярности довольно долгое время
    2) Вселенная с positive curvature (у нашей она равна нулю насколько я знаю) эквивалентна ЧД с очень большой массой и оч большим горизонтом. Причина — «закольцованность» Вселенной и невозможность выбраться за пределы горизонта.


    1. Victor_koly
      04.03.2019 18:04

      Вселенная могла бы быть закольцованной как минимум для массивных частиц материи, находящейся в ней (критическая плотность, кажется даже с учетом ТЭ, была весьма близка к реальной — 0.99..9 от критической). Но плотность оказалась ниже критической и через миллиарды лет Вселенная перешла (опять) на стадию расширения с ускорением.


  1. taujavarob
    04.03.2019 21:06

    Tzimie

    Более того, они так никогда и не упадут в черную дыру, ведь для этого нужно бесконечное время — ну, вы поняли что это совсем не так

    То есть — всего лишь сменой метрики, мы убеждаемся что черные дыры вполне могут образоваться? — Супер! — А нельзя ли таким «простым» манёвром доказать (или опровергнуть) почти все явления в ОТО?

    То есть, если что-то где-то в ОТО запрещено или «зависает», то «меняем метрику» и «проталкиваем» это куда-нибудь?

    Сменой метрики нельзя ли вообще убрать сингулярность в ОТО? Или «расширить проход» между кротовыми норами?


    1. Tzimie Автор
      04.03.2019 21:12

      Я там написал, что в запале перепутал слова — не метрики а системы координат.
      Бесконечность на горизонте устраняется сменой системы координат, а в центре ЧД — нет


      1. taujavarob
        05.03.2019 21:43

        Tzimie

        Бесконечность на горизонте устраняется сменой системы координат


        Раньше мы смотрели в телескоп и видели как всё вещество «зависает», бесконечно падая в ЧД, но всё упасть не может и поэтому мы решили что ЧД не существует.

        Теперь, мы смотрим в телескоп и видим, что хоть и происходит замедление падения, но оно НЕ бесконечно, — а за какое время внешний наблюдатель увидит как некая глыба вещества (звезда, пыль, корабль или космонавт) упала в ЧД?


        1. Tzimie Автор
          05.03.2019 22:02

          >мы смотрим в телескоп и видим, что хоть и происходит замедление падения, но оно НЕ бесконечно

          Не так. Замедление времени при свободном падении вообще не определено, так как траектория не замкнута, и падающий наблюдатель никогда не сможет снова синхронизовать часы

          Визуально же замедление бесконечно, но
          1. Это чисто оптический эффект
          2. Помимо замедления объект краснеет и блекнет, и в итоге пропадает из виду
          Поэтому ответ на ваш вопрос — никогда


    1. Tzimie Автор
      04.03.2019 21:14
      +1

      en.wikipedia.org/wiki/Schwarzschild_metric#Alternative_coordinates

      The singularity at r = rs is an illusion however; it is an instance of what is called a coordinate singularity. As the name implies, the singularity arises from a bad choice of coordinates or coordinate conditions. When changing to a different coordinate system (for example Lemaitre coordinates, Eddington–Finkelstein coordinates, Kruskal–Szekeres coordinates, Novikov coordinates, or Gullstrand–Painleve coordinates) the metric becomes regular at r = rs and can extend the external patch to values of r smaller than rs


  1. vadim_bv
    04.03.2019 22:08
    -1

    Кажется, это тот самый случай, когда заметка написана ради неё самой. Ничего экстраординарного я из неё не узнал (и про Интерстеллар тоже). Про черные дыры со всеми параметрами было написано еще в какой-то переводной книге (лень сейчас искать точное название, но в принципе могу), изданной еще в середине 1980-х (когда даже Хокинг еще не начал писать научпоп).


    1. Grigoriy_Chumakin
      05.03.2019 13:35
      +2

      Новиков Игорь Дмитриевич. Чёрные дыры и Вселенная. — М.: Молодая гвардия,
      1985. — 190 с., ил. — Серия «Эврика». Была такая дома в детстве)


      1. Tzimie Автор
        05.03.2019 13:35

        Помню ее)


      1. vadim_bv
        05.03.2019 22:03

        Раскопал свой школьный реферат по астрономии, который потом пересдал как допуск к кандидатскому минимуму :) и нашел там такие ссылки:
        И.Д. Новиков — в журнале «Земля и Вселенная» в 1970-80х. Наверное, то что было издано в серии «Эврика» — это компиляция из статей.
        Книга, про которую я говорю, это Иан Николсон «Тяготение, черные дыры и Вселенная». М., 1983.


  1. perfect_genius
    05.03.2019 10:00

    Вот что давно интересует: в дыру падает материя и свет, и на горизонте свет отскакивает от материи обратно и застревает на горизонте. Получается, чёрная дыра — «белый шар из света»? Но такое не происходит, потому что дыры не идеально стабильны, так? Да и материя не перестаёт ведь падать, постоянно снося этот слой из фотонов.


    1. Victor_koly
      05.03.2019 10:24
      -1

      Фотоны могут рассеиваться на плотной плазме вокруг горизонта. Если это плазма имеет элетронные свойства, близкие к металлу, то существенная часть фотонов опредленного диапазона длин волн может рассеяться наружу. Если эта плотность электронов высокая (скажем как плотность 1s-электронов в железе при н.у.) — может быдет сильно рассеиваться даже фотоны энергией до 1 МэВ.
      А при большей энергии фотоны начнут рождать электрон-позитронные пары, 1 из частиц пары упадет за горизонт (не путать с «правильным» и «неправильным» объяснением излучения Хокинга).


    1. thedeemon
      05.03.2019 13:36

      Если брать идеальное статичное решение (где размеры ЧД не меняются никогда), то да, формально горизонт событий это null surface, так же как границы световых конусов обычных. Свет, излученный на горизонте в сторону «наружу», остается на горизонте. На диаграмме Крускала это особенно хорошо видно.

      Там есть правда еще одна хитрость: поверхность эта не трехмерна (2д сфера х 1д время), а двухмерна, т.к. время на горизонте не тикает совсем. Что это значит физически — хороший вопрос. Но геометрически по ОТО выходит так.


      1. Victor_koly
        05.03.2019 13:46

        Как провести светоподобную траектории фотона по предельно измененной метрике на горизонте — такой вопрос у Вас получается?


  1. yul
    05.03.2019 10:26

    У меня остался один вопрос: с точки зрения внешнего наблюдателя, падающий объект застревает где-то на горизонте событий. Однако, в результате испарения ЧД, когда-нибудь горизонт событий начнёт уменьшаться, соответственно, наблюдатель будет видеть, что падающий объект приближается к центру ЧД, пока она не испарится полностью? То есть, так никогда и не пересечёт горизонт?


    1. 4tlen
      05.03.2019 11:00

      Свет от падающего в ЧД тела будет испытывать красное смещение. Поэтому сначала в видимом диапазоне он будет краснеть, затем перейдет в инфракрасный, затем в радиодиапазон. Так красным и растворится.


      1. Victor_koly
        05.03.2019 13:43

        Тут интересно рассмотреть такую границу «глубины источника фотонов». Пускай у нас на ЧД падают электрон и протон, которые в любой момент могут рекомбинировать и выдать фотон с энергией не ниже 13.5 эВ (то есть скажем если они и связаны изначально, то находятся на уровне не ниже n = 12).
        В какой-то момент испускается этот УФ фотон и пробует покинуть грав. колодец ЧД. Вопрос: «Начиная с какого растояния до горизонта (в какой бы метрике мы его не смогли измерить) за время отлета фотона на расстояние скажем 1.5 грав. радиусов от центра ЧД длина волны фотона превысит пройденный путь?».
        Или не может быть такого сильного красного смещения? Я конечно подозреваю, что такой «путь фотона» займет не один миллиард лет. Или можно заменить на условие «достигнет длины волны 1.5 грав. радиуса ЧД».


        1. 4tlen
          05.03.2019 14:34

          Или не может быть такого сильного красного смещения? Я конечно подозреваю, что такой «путь фотона» займет не один миллиард лет. Или можно заменить на условие «достигнет длины волны 1.5 грав. радиуса ЧД».
          Теоретически нет предела для длины волны при красном смещении, практически же фотон растворится в реликтовом. Это в идеализированной модели. На практике многое зависит от конкретной ситуации. Есть комптоновское рассеяние, которое ускорит снижение частоты фотона, есть обратный эффект Сюняева — Зельдовича (увеличение частоты фотонов за счет рассеяния на горячих электронах падающего в ЧД газа).


          1. Victor_koly
            05.03.2019 17:18

            По поводу горячих электронов и газа. Если фотон потеряет энергию до скажем 5 кэВ (в СО ЦМ с налетающим электроном), то очень слабо сможет изменить свою энергию за каждый акт комптоновского рассеяния.
            А если Вы говорите про реликтовое — фотонов конечно во Вселенной много (в миллиард раз больше, чем электронов, протонов и нейтронов?), но сколько из них долетят из сферы радиусом 15 Мпк до конкретной ЧД, расположенной в конкретной точке скажем галактического диска спиральной галактики?


    1. thedeemon
      05.03.2019 13:42

      Скорее, пересечет и исчезнет одновременно с исчезновением самой ЧД в ее финальной вспышке. См. диаграммы Пенроуза для испарающихся дыр. Например fias.uni-frankfurt.de/~hossi/Bilder/BR/bhevap.jpg
      (если забить на красное смещение)


  1. stanislavkulikov
    05.03.2019 14:14

    И всё-таки, можно поподробнее рассмотреть теорию космологии чёрной дыры, по которой наша 4-х мерная вселенная — это ЧД в 5-ти мерной вселенной. Она отлично согласуется со свёрнутыми измерениями в Теории Струн и объясняет множество парадоксов, но её почему-то пытаются проработать лишь несколько человек. Почему так?


    1. vedenin1980
      05.03.2019 15:06

      Почему так?

      Скорее всего потому, что ее никак не возможно проверить эксперементально и вряд ли будет когда-либо возможным. Ну сделали модель такой космологии, а дальше что? Все равно это никак не проверяется и не отвечает на вопрос, как 5-ти мерная вселенная выглядит.

      Потенциально она никак не отличается от теории, что наша Вселенная это шар в космическом бильярде. Таким теорий может быть сотни и все они одинаково неопровергаемые и непроверяемые.


      1. Victor_koly
        05.03.2019 17:22

        А может не шар в бильярде, а фермион (кварк или лептон, желательно заряженный и стабильный) в каком-то атоме «вселенной верхнего уровня». Очень старая теория, если что.


        1. Sdima1357
          05.03.2019 18:10

          Не обязательно верхнего, забавней если симметрично, наша вселенная у них частица, а их вселенная — у нас частица. Топология не запрещает :)


          1. Victor_koly
            05.03.2019 19:22
            +1

            На самом деле, это не я придумал. И вверх и вниз движение такого масштаба читал в этой книге:
            Древо жизни
            Где-то там была идея, конечно сюжет уже толком не помню. Раньше казалось, что книга из 3 частей, где 2я описывает жизнь в аналоге советского концлегеря.


  1. vlad9486
    05.03.2019 20:22

    Забавное совпадение. Первое решение уравнений ОТО описывающее черные дыры нашел человек с фамилией Шварцшильд (Schwarzschild), а «schwarz schild» с немецкого переводится как «черный щит».